<<

1. Consider the following statements (2) It is administered under Article 239 regarding the recent verdict of the AA which was incorporated in the Supreme Court on ’s demand Constitution of in 1992. It for statehood: created a “special” constitutional (1) Entry 41 in the State List set up for Delhi. (services; service (3) It has provisions for popularly commissions) would not be elected Legislative Assembly, a applicable to the Union Council of Ministers responsible to Territories (UTs) including the Legislative Assembly and a Delhi. certain demarcation of (2) As per Article 239 AA (3)(a) responsibilities between the LG and of the Constitution, the Delhi the Council of Ministers. Legislative Assembly can (4) As per Article 239 AA (3) (a), the legislate on all those matters Delhi Legislative Assembly can listed in the State List and legislate on all those matters Concurrent List as are listed in the State List and applicable to the Union Concurrent List as are applicable Territories. to the Union Territories. Which of the statements given (5) The public order, police and land above is/are correct? are reserved for the Lieutenant (a) 1 only Governor (LG). (b) 2 only Source: (c) Both 1 and 2 https://economictimes.indiatimes.com (d) Neither 1 nor 2 /news/et-explains/decoding-delhis- full-statehood- Answer: (b) demand/articleshow/64834484.cms Explanation: Statement 1 is incorrect: 2. LIMBS is often in the news Only three subjects are prohibited from regarding the Indian judiciary and government access – and ‘services’ is its governance. Consider the not one of them. Entry 41 of the State following statements in this context: List contains the subject of ‘services’. (1) It is an initiative of the Law The Constitution provides for the clear Commission of India. responsibility of this to the Delhi (2) It aims to make government government. Under the Government of litigation process more National Capital Territory of Delhi efficient and transparent. (GNCT) Act, 1991; Section 41, only Which of the statements given those matters outside the purview of the above is/are correct? government can be taken up by the (a) 1 only Lieutenant Governor (LG) and services is (b) 2 only not one of them. (c) Both 1 and 2 Statement 2 is correct: (d) Neither 1 nor 2 Article 239: (1) Delhi, although a Union Territory, is Answer: (b) not administered by the President Explanation: acting through the Lieutenant The government is a major litigant and it Governor (LG) under Article 239. is a party to about 46% of the 13.4 crore

Prelim IAS Test Series (2019) – GS Test 20 (19.05.2019) 1 GS Comprehensive Test 2 cases pending in various courts in the country, ranging from service matters to 3. Recently, the bomb indirect taxes. cyclone/bombogenesis swept Statement 1 is incorrect. Legal through the US Great Plains Information Management & Briefing resulting in blizzard conditions. System (LIMBS) is one of the Regarding ‘bomb cyclone’ consider progressive steps taken by the Ministry the following statements: of Law and Justice in the direction of (1) It refers to a rapidly digital India. It is based on the objective intensifying area of high- of achieving ‘minimum government and pressure winter storm. maximum governance. (2) It can be tropical or non- Statement 2 is correct: LIMBS helps in tropical. reducing government litigations; reduces (3) It is caused by a collision of the financial burden, saves time warm and cold air masses. and brings efficiency and introduces not (4) It is the rarest of rare only transparency but also a sense of phenomenon and occurs ownership among various stakeholders once in a decade. during the life cycle of a court case. Which of the statements given Value Addition: above are correct? Advantages of LIMBS: (a) 1 and 3 only  Organizing otherwise scattered (b) 2 and 3 only Information at one single database (c) 1 and 4 only and also create a professional (d) 1, 2 and 4 only base available for expert advice  A low cost web technology access Answer: (b) to all the stakeholders involved in a Explanation: court case in a coordinated  The term bombogenesis is a popular way whereby it provides inputs which term used by meteorologists to refer are available seamlessly on 24×7 to a rapidly intensifying area of low basis as per the defined access pressure. The central pressure of an rules. area of low pressure [winter storm]  In this way it is possible to know the must drop at least 24 millibars in 24 various stages of a case and monitor hours to qualify as a cyclone. Hence, its progress on a continued basis Statement 1 is incorrect. through an elaborate set of user-  A bombogenesis storm can be friendly reports. tropical or non-tropical. Hence,  It will help authorities to take ‘data Statement 2 is correct. driven decision making’ and to  Bomb Cyclone is caused by a evaluate the performance of various collision of warm and cold air masses stake holders and to conduct a legal which develop into a rotating storm- audit. like pattern and lead to an explosive Source: deepening of pressure. Hence, http://pib.nic.in/newsite/PrintRelease.a Statement 3 is correct. spx?relid=146572  It’s not unusual to get at least one storm that is classified as a bomb cyclone a year. Hence, Statement 4 is incorrect.

Prelim IAS Test Series (2019) – GS Test 20 (19.05.2019) 2 GS Comprehensive Test 2

Source: and promoting coastal community https://www.foxnews.com/us/what-is- development, with the aim of boosting a-bomb-cyclone merchandise exports by US$110 billion and generating around 10,000,000 direct 4. Consider the following statements: and indirect jobs. (1) The Sagarmala programme is the flagship programme of 5. Consider the following statements the Ministry of Shipping to in the context of Aadhaar: promote port-led (1) The Supreme Court struck development in the country. down Section 57 of the (2) Coastal Community Aadhaar Act which allowed Development through skill private entities to use development is one of the Aadhaar for verification strategies adopted in the process. Sagarmala programme. (2) Sharing of data with the Which of the statements given specially authorized officers above is/are correct? in interest of national security (a) 1 only is validated by the verdict. (b) 2 only (3) Linking of the Aadhaar with (c) Both 1 and 2 Permanent Account Number (d) Neither 1 nor 2 (PAN) is necessary. Which of the statements given Answer: (c) above is/are correct? Explanation: (a) 1 only Statement 1 is correct. The Sagarmala (b) 2 and 3 only Programme is the flagship programme of (c) 1 and 3 only the Ministry of Shipping to promote port- (d) 1, 2 and 3 led development in the country by exploiting India’s 7,500 km long Answer: (c) coastline, 14,500 km of potentially Explanation: navigable waterways and its strategic Aadhaar: The recent Supreme Court location on key international maritime verdict: trade routes.  The Supreme Court has struck down Statement 2 is correct. The Sagarmala Section 57 of the Aadhaar Act, hence programme envisages unlocking the private companies can no longer ask potential of waterways and the coastline for a consumer's Aadhaar details. to minimize infrastructural investments Hence, statement 1 is correct. required to meet these targets. It entails  Sharing of data with the specially investing ₹8.5 trillion (equivalent to ₹8.9 authorized officers in interest of trillion, US$120 billion or €110 billion in national security is not validated by 2018) to set up new mega ports, the verdict. The apex court asked the modernizing India's existing ports, Central government to introduce a developing of 14 Coastal Economic strong data protection law as soon as Zones (CEZs) and Coastal possible to deal with the matter of Employment Units, enhancing port data sharing. Hence, statement 2 is connectivity via road, rail, multi-modal incorrect. logistics parks, pipelines & waterways

Prelim IAS Test Series (2019) – GS Test 20 (19.05.2019) 3 GS Comprehensive Test 2

 The linkage of Aadhaar and Which of the statements given Permanent Account Number (PAN) above are incorrect? remains mandatory. PAN is a 10-digit (a) 1, 2 and 3 only alphanumeric number issued to (b) 1, 3 and 4 only Assessee by the Income Tax (c) 1, 2 and 4 only Department and is mandatory for (d) 2, 3 and 4 only filing income tax returns. Hence, statement 3 is correct. Answer: (b) Additional Information: Explanation:  The need to link Customers bank Statement 1 is incorrect. MUDRA account with the Aaadhar number (Micro Units Development & Refinance has been done away with. Agency Ltd) is a financial institution being  Users no longer need to produce set up by Government of India for their Aadhaar number for procuring development and refinancing micro units’ the SIM cards enterprises. It was announced by the  The 12-digit unique identification Hon’ble Finance Minister while number need not be produced at the presenting the Union Budget for FY 2016 th time of school admissions or (during 12 Five Year Plan). The enrolment in exams of NEET, CBSE purpose of MUDRA is to provide funding and UGC to the non-corporate small business  The apex court also asserted that the sector through various Last Mile Aadhaar card will not be provided to Financial Institutions like Banks, NBFCs, illegal migrants. and MFIs. Source: Statement 2 is correct. The https://www.livemint.com/Politics/M5yPb Government of India set up MUDRA Ml8duh31IHVUv0lPP/Aadhaar-supreme- Bank through a statutory enactment for court-verdict-uidai-bank-account-mobile- catering to the needs of the informal link.html sector for bringing them into the mainstream economy. Pending 6. Consider the following statements enactment of an Act for the MUDRA regarding the MUDRA Bank: Bank, a Non-Banking Finance Company (1) It was established during the (NBFC) as MUDRA Ltd has been set up 11th Five-Year Plan to as a subsidiary of the SIDBI. facilitate credit to Small and Statement 3 is incorrect. MUDRA Medium Entreprises. would be responsible for refinancing all (2) It is a Non Banking Finance Last Mile Financiers such as Non- Company set up as a Banking Finance Companies, Societies, subsidiary of the Small Trusts, Section 8 Companies [formerly Industries Development Bank Section 25], Co-operative Societies, of India. Small Banks, Scheduled Commercial (3) It directly provides loans to Banks and Regional Rural Banks which non-corporate small business are in the business of lending to sector of up to Rs. 10 lakhs. micro/small business entities engaged in (4) Service sector activities are manufacturing, trading and services not eligible for availing activities. MUDRA would also partner MUDRA loan. with State/Regional level financial intermediaries to provide finance to Last

Prelim IAS Test Series (2019) – GS Test 20 (19.05.2019) 4 GS Comprehensive Test 2

Mile Financier of small/micro business  Philippines announced to withdraw enterprises. It doesn’t provide credit from International Criminal Court directly. (ICC) citing reason of international Statement 4 is incorrect. All kinds of bias. manufacturing, trading, and service Source: https://www.business- sector activities can get MUDRA loan. standard.com/article/pti- Loans are categorized into Shishu, stories/malaysia-joins-icc-in-boost- Kishor, and Tarun. for-beleaguered-court- 119030500257_1.html 7. Which of the following countries has recently announced to join the 8. The UPSC consists of a Chairman International Criminal Court (ICC)? and other members appointed by (a) Philippines the President of India. In this (b) Malaysia context, which of the following (c) Pakistan statements is incorrect? (d) Sri Lanka (a) The President determines the strength and composition of Answer: (b) the Commission. Explanation: (b) The Constitution authorises  The International Criminal Court the President to determine (ICC) is the world’s first the conditions of service of intergovernmental legal body with the members of the permanent international jurisdiction Commission. to prosecute individuals for genocide, (c) No qualifications are crimes against humanity and war prescribed for the crimes. Commission’s membership in  It is based in the Hague, the Constitution. Netherlands. India is not a member (d) The entire expenses of the of the ICC. US signed the treaty UPSC are charged upon the (Rome Statute) in 2000 but never Consolidated Fund of India. ratified it, citing concerns over sovereignty, similarly is the case with Answer: (c) Russia. Israel signed it for a short Explanation: period but also never ratified it into The UPSC consists of a Chairman and law. China has not signed it. other members appointed by the  The ICC has jurisdiction to prosecute President of India. individuals and is independent of  The Constitution, without specifying United Nations (UN). But it may the strength of the Commission has receive case referrals from the UN left the matter to the discretion of the Security Council and can initiate President, who determines its prosecutions without UN action or composition. referral.  Usually, the Commission consists of  Malaysia has ratified the Rome nine to eleven members including Statute making it the 124th State the Chairman. party to the International Criminal  Further, no qualifications are Court (ICC). prescribed for the Commission’s membership except that one-half of

Prelim IAS Test Series (2019) – GS Test 20 (19.05.2019) 5 GS Comprehensive Test 2

the members of the Commission Value Addition: should be such persons who have The Public Affairs Index (PAI) covers held office for at least ten years either wide range of themes such as support to under the Government of India or human development, social protection, under the government of a State. essential infrastructure, women and  The Constitution also authorises the children, crime, law and order, delivery of President to determine the conditions justice, transparency and accountability, of service of the Chairman and other environment, fiscal management and members of the Commission. economic freedom. PAC has added  The entire expenses of the UPSC are ‘Children of India’ as an additional charged on the Consolidated Fund of measurement parameter in 2018. India. Source: http://pai.pacindia.org/ Hence, the correct answer is option (c). 10. Consider the following statements: Source: Indian Polity by M (1) The International Labour Lakshmikanth; 5th Edition, Chapter 43 Organisation (ILO) was Pg-772 created in 1919 as a part of the Treaty of Versailles that 9. In the context of the ‘Public Affairs ended the World War I. Index’, consider the following (2) Being pre-United Nations statements: body, the International (1) Kerala has topped the index Labour Organization is not for the second time in a row. one of the United Nations (2) This index is released by the agencies. Department of Administrative Which of the statements given Reforms, Ministry of above is/are correct? Personnel, Public Grievances (a) 1 only and Pensions annually. (b) 2 only Which of the statements given (c) Both 1 and 2 above is/are correct? (d) Neither 1 nor 2 (a) 1 only (b) 2 only Answer: (a) (c) Both 1 and 2 Explanation: (d) Neither 1 nor 2 Statement 1 is correct. The International Labour Organisation (ILO) Answer: (a) was created in 1919 as a part of the Explanation: Treaty of Versailles that ended the World Statement 1 is correct: In the Public War I. The ILO will celebrate its 100th Affairs Index (PAI) 2018, Kerala topped anniversary in 2019. the list for the second time in a row as Statement 2 is incorrect. The the best-governed state in the country International Labour Organization (ILO) followed by Tamil Nadu. is a United Nations agency whose Statement 2 is incorrect: The index is mandate is to advance social justice and released since 2016 by Bengaluru based promote decent work by setting Public Affairs Centre (PAC), a not for international labour standards. It was the profit think tank which aims to improve first specialised agency of the UN. The governance in India. ILO has 187 member states: 186 of the

Prelim IAS Test Series (2019) – GS Test 20 (19.05.2019) 6 GS Comprehensive Test 2

193 UN member states plus the Cook 12. Consider the following statements Islands (South Pacific) are members of about Alternative Dispute the ILO. The tripartite structure is unique Resolution (ADR) mechanisms: to the ILO where representatives from (1) Arbitration is the process of the government, employers and hearing and determining employees openly debate and create disputes between parties by labour standards. persons chosen by them to obtain fair resolution of 11. The gravitation force (g) is not the disputes by an impartial same at different latitudes on the tribunal. Earth's surface. In this context (2) Reconciliation is a process of consider the following statements: facilitating amicable (1) It is greater near the equator settlement between parties, and less at the poles. there may not be (2) The gravity values differ determination of dispute and according to the mass of the parties may not be forced to material. accept it. Which of the statements given (3) Mediation aims to assist two above is/are correct? or more disputants in (a) 1 only reaching an agreement, the (b) 2 only parties themselves decide (c) Both 1 and 2 conditions of the settlement. (d) Neither 1 nor 2 Which of the statements given above are correct? Answer: (b) (a) 1 and 2 only Explanation: (b) 2 and 3 only Statement 1 is incorrect. The (c) 1, 2 and 3 gravitation force (g) is not the same at (d) None of the above different latitudes on the Earth's surface. It is greater near the poles and less at Answer: (c) the equator. This is because of the Explanation: All the given statements distance from the centre at the equator are correct. being greater (due to Earth’s bulging out Alternative Dispute Resolution (ADR) is at equator) than that at the poles. an out of Court settlement which can Statement 2 is correct. The gravity be discussed by any party at any time values also differ according to the mass during litigation and is often a cost- of material. The uneven distribution of effective alternative to trial. They are mass of material within the Earth being promoted in a big way in view of influences this value. The reading of the the huge arrears of the cases and delay gravity at different places is influenced by in justice delivery. many other factors. These readings differ  Arbitration is a traditional alternative from the expected values. Such a to the Court-based litigation which is difference is called gravity anomaly. often quite time and resource Gravity anomalies give us information consuming. Arbitration is a quasi- about the distribution of mass of the judicial process in which there are material in the crust of the Earth. two disputing parties and a third, neutral person, called as arbitrator or

Prelim IAS Test Series (2019) – GS Test 20 (19.05.2019) 7 GS Comprehensive Test 2

arbiter, who sits as a private judge. It states of the UN General is binding method of dispute Assembly. resolution governed by Statute. The Which of the statements given arbitrator resolves the dispute of the above is/are correct? parties in a confidential manner. (a) 3 only  Conciliation is an alternative dispute (b) 1 and 2 only resolution process whereby a neutral (c) 2 and 3 only third party is appointed as a (d) 1 and 3 only Conciliator with the consent of disputants. The Conciliator is not Answer: (a) bound by the rules of evidence. Explanation: His job is to pacify the two parties by Statement 1 is incorrect: National Hindi setting their issues. This Conciliator Divas and World Hindi day are entirely then requests both the parties to different. National Hindi Diwas was first prepare a list of objectives they wish celebrated on September 14 in 1949 to resolve. At no point of time during when the Constitution of India declared the Conciliation process do the two Hindi written in Devanagari Script as an parties meet. official language. The day is observed  Mediation is a voluntary and within the country only. On the other consensual process where in the hand, the World Hindi Day is observed to disputing parties are assisted in promote the greatness of Hindi language reaching a mutually agreeable at the global level. The first World Hindi settlement by a neutral third party Conference was held on January 10 in termed as mediator, whose role is to 1975. To commemorate this special day, facilitate the communications and World Hindi Day is celebrated on the discussions, same date every year. Source: Statement 2 is incorrect: Recently, https://www.thehindu.com/opinion/op- United Arab Emirates (UAE) has ed/encouraging-mediation-to-settle- declared Hindi as its third official disputes/article24273149.ece language. So, India is not the only country in the world having Hindi as its 13. Consider the following statements official language. with respect to the Hindi Language: Statement 3 is correct: The question of (1) World Hindi day, which is addition of any language including Hindi observed to promote the in the list of official languages of the greatness of Hindi language United Nations has procedural, financial at the global level, is and legal aspects. As a first step, a celebrated on the day of its formal resolution needs to be adopted adoption as an official by a majority of the 193 member states language in India. of the UN General Assembly. No (2) India is the only country in resolution has been introduced in this the world having Hindi as an regard as yet. official language. Sources: (3) The adoption of Hindi by  http://pib.nic.in/newsite/PrintRelea United Nations as an official se.aspx?relid=107674 language requires support of  http://www.newindianexpress.com/ majority of 193 member world/2019/feb/12/uae-introduces-

Prelim IAS Test Series (2019) – GS Test 20 (19.05.2019) 8 GS Comprehensive Test 2

hindi-as-third-official-language- (2) It has high concentration of 1937849.html the oxidizing agents and so  https://www.financialexpress.com/i is called as oxidizing smog. ndia-news/world-hindi-day-2019- (3) It is formed from the action of why-world-hindi-day-is-observed- sunlight on unsaturated on-january-10-check-facts-and- hydrocarbons and nitrogen significance/1439383/ oxides. Which of the statements given 14. Consider the following statements: above is/are correct? (1) The AMRUT mission was (a) 1 only launched to establish (b) 1 and 2 only adequate infrastructure in the (c) 2 and 3 only rural areas. (d) 1, 2 and 3 (2) The AMRUT mission is being implemented by the Ministry Answer: (c) of Rural Development. Explanation: Which of the statements given Statement 1 is incorrect. above is/are correct? Photochemical smog is a type of smog (a) 1 only produced when ultraviolet light from the (b) 2 only sun reacts with nitrogen oxides in the (c) Both 1 and 2 atmosphere. It is visible as a brown (d) Neither 1 nor 2 haze, and is most prominent during the morning and afternoon, especially in Answer: (d) densely populated, warm cities. Hotter Explanation: days mean more photochemical smog, Statement 1 is incorrect. The especially in the densely populated cities. Government of India has launched the Statement 2 is correct. Photochemical Atal Mission for Rejuvenation and Urban smog is also sometimes known as Transformation (AMRUT) with the aim of "oxidizing smog", in that it has a high providing basic civic amenities like water concentration of oxidizing agents. Ozone supply, sewerage, urban transport, parks is a common oxidizing agent found in as to improve the quality of life for all photochemical smog. especially the poor and the Statement 3 is correct. Three disadvantaged in the urban areas. The ingredients - energy from a light source focus of the Mission is on infrastructure (ultraviolet), unsaturated hydrocarbons, creation that has a direct link to provision and nitrogen oxides - are needed for of better services to the citizens. photochemical smog to be formed. Two Statement 2 is incorrect. The mission is of those components are produced being implemented by the Ministry of through the burning of fossil fuels, most Housing and Urban Affairs. notably automobiles.

15. Consider the following statements 16. Consider the following statements about photochemical smog: regarding the North East Council: (1) Photochemical smog occurs (1) The North East Council is a in cool humid climate. ‘non-statutory’ body established in 1971.

Prelim IAS Test Series (2019) – GS Test 20 (19.05.2019) 9 GS Comprehensive Test 2

(2) After the 2002 amendment, it (3) It will ensure the enforcement has been made a regional of Right to Life in its spirit. body for the North-East Which of the statements given region. above is/are correct? (3) It operates under the (a) 1 only administrative control of the (b) 2 and 3 only Home Ministry. (c) 1 and 3 only Which of the statements given (d) 1, 2 and 3 above is/are correct? (a) 1 only Answer: (c) (b) 2 only Explanation: (c) 1 and 2 only Statement 1 is correct: Always being (d) 2 and 3 only connected has been taking a toll on workers the world over, studies show. Answer: (b) Various studies found that even if Explanation: workers aren’t actively checking work Statement 1 is incorrect: The North emails after hours, they can still be Eastern Council (NEC) is not a harmed by the expectation that they constitutional body, but a statutory should be available. It has been found organization established under the North that that this kind of “always on” work Eastern Council Act, 1971, as amended culture creates anxiety amongst workers. in 2002. Statement 2 is incorrect: The Article 7 Statement 2 is correct: Initially, the of the Right to Disconnect Bill says that: NEC was an Advisory Body for the North Every employee shall have the right to Eastern Region (NER). Now, the NEC is disconnect out of work hours. However mandated to function as a Regional the 'right to disconnect' means that Planning Body for the North Eastern while the employer may contact the Region. The Council comprises worker after work hours, the employee Governors and Chief Ministers of is not obliged to reply or shall have constituent States and three members right to refuse to answer such calls; nominated by the President. and (b) In case an employee refuses to Statement 3 is incorrect: It operates reply any call during out-of-work hours, under the Ministry of Development of such employee shall not be subject to North-Eastern Region (DoNER). any disciplinary action by the employer Source: Statement 3 is correct: In Bandhua http://pib.nic.in/newsite/PrintRelease.a Mukti Morcha v. Union of India spx?relid=181418 characterizing Article 21 of the Indian Constitution as the heart of fundamental 17. Consider the following statements rights, the Court gave it an expanded with respect to the ‘Right to interpretation. Bhagwati J. observed: “It Disconnect Bill’: is the fundamental right of everyone in (1) It is intended to help reduce this country… to live with human dignity work related stress and strive free from exploitation. This right to live for a better work-life balance. with human dignity enshrined in Article (2) It debars the employer to 21 derives its life breath from the contact the employee after Directive Principles of State Policy work hours. and particularly clauses (e) and (f) of

Prelim IAS Test Series (2019) – GS Test 20 (19.05.2019) 10 GS Comprehensive Test 2

Article 39 and Articles 41 and 42 of the Statement 2 is incorrect: World Bank Constitution of India. will fund the project. The Inland Sources: Waterways Authority of India (IWAI) has  https://www.lawctopus.com/acade been assigned the task of executing this mike/article-21-of-the-constitution- project. of-india-right-to-life-and-personal- Statement 3 is correct. The pilot of the liberty/ project will be in Varanasi and after that  http://164.100.24.219/BillsTexts/LS more villages will connect freight BillTexts/AsIntroduced/2317as.pdf corridors in the country.  https://economictimes.indiatimes.c om/jobs/this-bill-will-give-you-the- 19. Consider the following statements: right-to-ignore-your-boss-after- (1) When the pH of rain water work/how-will-it- drops below 7 it is called as work/slideshow/67510867.cms acid rain. (2) Oxides of nitrogen and 18. With reference to the 'Freight carbon, after reaction with Village', consider the following rainwater are major statements: contributors to acid rain. (1) It is a specialized cluster of Which of the statements given villages dedicated to the above is/are correct? manufacturing sector. (a) 1 only (2) World Bank will fund the (b) 2 only project and the NITI Aayog (c) Both 1 and 2 has been assigned the task (d) Neither 1 nor 2 of executing this project. (3) Varanasi will get India’s first Answer: (d) freight village. Explanation: Which of the statements given Acid rain, or acid deposition, is a broad above is/are correct? term that includes any form of (a) 3 only precipitation with acidic components, (b) 1 and 2 only such as sulfuric or nitric acid that fall to (c) 1, 2 and 3 the ground from the atmosphere in wet (d) None of the above or dry forms. This can include rain, snow, fog, hail or even dust that is acidic. Answer: (a) Statement 1 is incorrect. Normal, clean Explanation: rain has a pH value of between 5.0 and Statement 1 is incorrect. Freight village 5.5, which is slightly acidic. However, is a specialized industrial estate to attract when rain combines with sulfur dioxide or companies in need of logistics services nitrogen oxides—produced from power and can cluster to improve their plants and automobiles—the rain competitiveness. The concept will also becomes much more acidic. Typical acid bring together retailers, warehouse rain has a pH value of 4.0. operators and logistics service providers Statement 2 is incorrect. Acid rain supplying the regional FMCG market. It results when sulfur dioxide (SO2) and is modelled on concepts that are nitrogen oxides (NOX) are emitted into prevalent in Europe. the atmosphere and transported by wind and air currents. The SO2 and NOX

Prelim IAS Test Series (2019) – GS Test 20 (19.05.2019) 11 GS Comprehensive Test 2 react with water, oxygen and other created by an Act of Parliament. It chemicals to form sulfuric and nitric consists of both elected and nominated acids. These then mix with water and members. other materials before falling to the Town Area Committee: A town area ground. committee is set up for the administration of a small town. It is a semi-municipal 20. Which of the following pairs are authority and is entrusted with a limited correctly matched? number of civic functions like drainage, Types of Nature/Funct roads, etc. It is created by a separate act Urban ions of the State Legislature. Governm Special Purpose Agency: The state ents have set up certain agencies to 1. Port Trust : Created by an undertake designated activities or Act of the specific functions that legitimately belong Parliament to to the domain of municipal corporations manage and or municipalities or other local urban protect ports governments. These are function based 2. Town : Semi- and not area based. Area municipal Source: Vajiram and Ravi Yellow Book Committe authority for Indian Polity Part 2: Chapter 1 e the Government of the State; page no 5 administration of a small 21. Recently, which of the following town Indian State granted religious 3. Special : Autonomous minority status to the Jews? Purpose bodies (a) Gujarat Agency created (b) Goa independently (c) Kerala of local (d) Karnataka governments to undertake Answer: (a) specific Explanation: functions. Significance: Select the correct answer using  The Gujarat Government has granted the code below: religious minority status to the Jews (a) 1 and 2 only living in the State. (b) 2 and 3 only  Gujarat is the third state in India to (c) 1 and 3 only grant religious minority status to Jews (d) 1, 2 and 3 after West Bengal and Maharashtra. Answer: (d) Hence, option (a) is the correct Explanation: All the pairs given above answer. are matched correctly. Additional Information: Port Trust: These are established in the  Judaism is one of the oldest religions port areas for two purposes: (a) to of the world, evolved in Egypt about manage and protect ports; and (b) to 3,700 years ago. It believes in the provide civic amenities. A port trust is unity and oneness of universal

Prelim IAS Test Series (2019) – GS Test 20 (19.05.2019) 12 GS Comprehensive Test 2

Creator. Judaism is the religion, (1) Treasury Bill is an instrument philosophy and way of life of the issued by the government to Jewish people. Jews have been living finance its short-term in India for over 2,000 years ever requirements. since they first landed on West coast (2) Treasury Bill is one of the of India. safest instruments to invest  Indian Jews are known as a peace- since they are issued by the loving community. They follow government. Hebrew calendar. They have special Which of the statements given thanks giving ceremony known as above is/are correct? Eliyahoo-ha-Nabior i.e. ‘gratitude (a) 1 only to Elijah the Prophet’, on festive (b) 2 only occasions. (c) Both 1 and 2  Indian Jews fall into five (d) Neither 1 nor 2 categories (1) Bene Israel – meaning Children Answer: (c) of Israel. Marathi speaking. Explanation: Arrived in Maharashtra 2,100 Both the statements are correct. years ago. Treasury-Bills are zero coupon securities (2) Cochin Jews – arrived in India that are issued at a discount and 2,500 years ago and settled down redeemed in face value at maturity. in Kerala as traders. These are issued to address short term (3) Baghdadi Jews – Jews who receipt-expenditure mismatches under came to India as traders from the auction program of the Government. West Asia, mainly from Baghdad. These are primarily issued in three They are settled mainly in tenors, 91,182 and 364 day. Treasury Mumbai, Pune and . bills are one of the safest instruments to (4) Bene Menashe – The Manipur invest since they are issued by the Jews constitute a community government. Recently the Reserve Bank which sees itself as descendants of India has allowed Foreign Portfolio of the Manasseh (Menashe) Tribe Investors to invest in Treasury Bills (T- (which is one of the 10 lost tribes Bills). of Jews). (5) Bene Ephraim – also called 23. Which of the following statements “Telugu Jews”. They are a small regarding ‘insulin hormone’ is group who speak Telugu. Their correct? observance of Judaism dates to (a) Insulin is a protein hormone 1981. which is produced by liver in Source: human body. https://timesofindia.indiatimes.com/in (b) Insulin is a steroid hormone dia/gujarats-jewish-community-gets- which is produced in human religious-minority- body. status/articleshow/64921599.cms (c) Insulin is a cholesterol-driven hormone given to humans. 22. In the context of 'Treasury Bill', (d) Insulin is a protein hormone which was in the news recently, which is produced by consider the following statements: pancreas in human body.

Prelim IAS Test Series (2019) – GS Test 20 (19.05.2019) 13 GS Comprehensive Test 2

Law Commissions have so far submitted Answer: (d) 262 reports. Explanation: Statement 1 is incorrect: The Law Option (d) is the correct answer. Commission of India is a non-statutory Insulin is an essential hormone produced body constituted by the Government of by the pancreas. Its main role is to India from time to time. The Law control glucose levels in our bodies. Commission was originally constituted in Insulin is a protein chain or peptide 1955 and is re-constituted very three hormone. years. Steroid hormones are secreted by three Statement 2 is correct: Cabinet gave endocrine organs: the testes, which approval to constitute the 21st Law produce testosterone; the ovaries, which Commission of India for a period of 3 produce oestrogen; and the adrenal years (2015-2018). The tenure of the cortex, which produces steroid 20th Law Commission was upto 31st hormones such as cortisol and August, 2015. aldosterone. Cholesterol is the parent Statement 3 is correct: The Law compound from which steroids are Commission of India is an advisory body derived. Therefore, these hormones are whose recommendations are not binding also called as cholesterol-driven on the government. hormones. Value Addition: Composition 24. Consider the following statements:  a full-time Chairperson; (1) The Law Commission of  four full-time Members (including a India is a statutory body Member-Secretary); constituted by the Union  Secretary, Department of Legal Government from time to Affairs as ex officio Member; time.  Secretary, Legislative Department as st (2) The 21 Law Commission of ex officio Member; and India had been formed to  Not more than five part-time function for a period of 3 Members. years. Functions (3) It is an advisory body whose  Undertake research in law recommendations are not and review of existing laws in India binding on the government. for making reforms therein and Which of the statements given enacting new legislations. above are correct?  Undertake studies and research for (a) 1 and 2 only bringing reforms in the justice (b) 2 and 3 only delivery systems for elimination of (c) 1 and 3 only delay in procedures, speedy disposal (d) None of the above of cases, reduction in cost of litigation etc. Answer: (b)  Identify laws which are no longer Explanation: relevant and recommend for Various Law Commissions have been the repeal of obsolete and able to make important contribution unnecessary enactments. towards the progressive development and codification of laws of the country.

Prelim IAS Test Series (2019) – GS Test 20 (19.05.2019) 14 GS Comprehensive Test 2

 Suggest enactment of new become permanent government legislations as may be necessary to employees. implement the Directive Principles Statement 2 is incorrect: The Article 16 and to attain the objectives set out in of the Indian Constitution states that the Preamble of the Constitution. “There shall be equality of opportunity for  Prepare and submit to the Central all citizens in matters relating to Government, from time to time, employment or appointment to any office reports on all issues, matters, studies under the State”. However clause 4 of and research undertaken by it and the same Article also empowers the state recommend in such reports for to make provisions for the reservation of effective measures to be taken by the appointments or posts in favour of any Union or any State backward class of citizens which, in the Source: opinion of the State, is not adequately http://lawcommissionofindia.nic.in/ represented in the services under the State. At the same time the Supreme 25. Consider the following statements Court has also observed that in its with respect to the ‘One Family, ordinary connotation the expression One Job’ scheme: 'class' means a homogenous section of (1) The Sikkim government the people grouped together because of envisages employment to a certain likenesses or common traits, and member, of such families of who are identifiable by some common the State, who does not have attributes such as status, rank, a government job. occupation, residence in a locality, race (2) It amounts to violation of the religion and the like". Thus, the scheme Article 16 of the constitution, is not a violation of Article 16. which advocates equality in Sources: matters of public  http://persmin.gov.in/otraining/UN employment. DPProject/undp_modules/Reservat Which of the statements given ion%20in%20Service.pdf above is/are correct?  https://www.business- (a) 1 only standard.com/article/current- (b) 2 only affairs/rojgar-mela-2019-sikkim- (c) Both 1 and 2 cm-launches-one-family-one-job- (d) Neither 1 nor 2 scheme-119011200354_1.html

Answer: (a) 26. 'Kandhamal Haldi', a variety of Explanation: turmeric has earned the Statement 1 is correct: Sikkim Geographical Indication (GI) tag. Government has launched 'One Family, With reference to ‘Kandhamal One Job' scheme that envisions Haldi’, which of the following employment to a member of every family statements is/are correct? which does not have a government job in (1) It is indigenous to southern the State. Under the scheme the Karnataka. temporary/ad hoc appointments being (2) It gives strong aroma but has given now would be regularised over the a low medicinal value. next five years, and all of them would Select the correct answer using the code given below:

Prelim IAS Test Series (2019) – GS Test 20 (19.05.2019) 15 GS Comprehensive Test 2

(a) 1 only machinery, the government of the State (b) 2 only will not be functional and the State will (c) Both 1 and 2 come under the direct control of the (d) Neither 1 nor 2 Central/Union government through the Governor. Answer: (d) Statement 2 is correct: During the State Explanation: Emergency also, the President takes Statement 1 is incorrect. Kandhamal over the work of the State through the Haldi, a variety of turmeric is indigenous Governor. Until the point of time when to southern Odisha. Kandhamal the President's rule is in effect, the Haldi has more oleoresin and volatile Parliament makes laws on the 66 oil contents as compared to the other subjects made on the State list. The turmeric varieties. money bills of the State are also Statement 2 is incorrect. Kandhamal approved by the Parliament during this Haldi gives strong aroma and has a high time. medicinal value. Statement 3 is correct. When the Source: https://www.business- President's rule is imposed, the standard.com/article/economy- assembly either is dissolved or is policy/on-foundation-day-odisha- suspended for the time by the President. receives-gi-tag-for-kandhamal-haldi- The State government comes under the 119040101058_1.html direct control of the Centre with the executive authority being shifted to the 27. Which of the following is/are the Governor (As an agent of the Centre) consequences of ‘Constitutional from the Chief Minister and the Council Emergency’ as per the Indian of Ministers. Constitution? Source: (1) The State government comes https://www.indiatoday.in/education- under direct control of the today/gk-current-affairs/story/an- Union government. insight-to-the-territorial-powers-of- (2) The money bills of the State the-indian-president-248722-2015-04- are approved by the 15 Parliament of India. https://www.rediff.com/news/2005/mar (3) The Legislative Assembly of /15spec1.htm the State concerned is dissolved or suspended. 28. Consider the following statements Select the correct answer using the with reference to the Manjeera code given below: Wildlife Sanctuary: (a) 3 only (1) It is located in Maharashtra (b) 1 and 3 only state of India. (c) 2 and 3 only (2) It has a Manjeera reservoir (d) 1, 2 and 3 made on a tributary of the Godavari river. Answer: (d) Which of the statements given Explanation: above is/are correct? Statement 1 is correct: According to the (a) 1 only Constitution of India, Article 356 deals (b) 2 only with the failure of Constitutional (c) Both 1 and 2

Prelim IAS Test Series (2019) – GS Test 20 (19.05.2019) 16 GS Comprehensive Test 2

(d) Neither 1 nor 2 Explanation: Statement 1 is incorrect: With respect Answer: (b) to relatively well preserved stupa at Explanation: Sanchi, there were sculptural decorations Statement 1 is incorrect: Manjeera on railings and gateways. There were wildlife sanctuary is located in Telangana garlands decorating the stupas as State. represented in reliefs. The plaster over Statement 2 is correct: The Manjeera stupa and paint have eroded away with Wildlife Sanctuary has is a man-made time, that’s why they look austere today, reservoir, made on Manjeera river which but were not designed that ways. Same is a tributary of Godavari river. It also goes for Amravathi stupa as well. supplies drinking water to Hyderabad Statement 2 is correct: The shape of city. It is home to vulnerable species of the stupa represents Buddha, crowned mugger crocodiles, and also provides and sitting in meditation posture on a lion lush nesting ground for migratory birds. throne. His crown is top of the spire, and The Manjeera barrage and the Singur so on, and the base is his throne. reservoir going dry this summer, has Statement 3 is correct: The stupa forced the crocodiles from the Manjeera architecture represents the integration of Wildlife sanctuary to search for any place five elements as chakras in human form. where water is logged. It has been depicted in different manner Source:https://www.thehindu.com/news/ in Buddhist architecture of India, Tibet national/telangana/dry-singur-has-crocs- and Japan. searching-for-water/article26891434.ece Source: Vajiram and Ravi booklet on Indian Art and Culture, Volume 1, 29. Which of the statements given Chapter 2, Page 27, 28 and 34. below are correct with reference to the Stupa? 30. Consider the following statements (1) The stupa signified about the Rock-Cut Architecture in ‘detachment’; hence their Indian subcontinent: architecture was very (1) It is mostly religious in nature. conservative and austere. (2) There is a distinct influence of (2) The shape of the stupa timber use. represents Buddha, crowned (3) Rock cut Chaitya shrines and sitting in meditation were dedicated to the posture on a lion throne. Theravad School only. (3) The stupa architecture Which of the statements given represents the integration of above is/are correct? five elements as chakras in (a) 1 and 2 only human form. (b) 2 only Select the correct answer using the (c) 2 and 3 only code given below: (d) 3 only (a) 1 and 2 only (b) 1 and 3 only Answer: (a) (c) 2 and 3 only Explanation: (d) 1, 2 and 3 Statement 1 is correct: Indian rock-cut architecture is mostly religious is nature. Answer: (c) All the rock-cut caves belong either to

Prelim IAS Test Series (2019) – GS Test 20 (19.05.2019) 17 GS Comprehensive Test 2

Buddhism, Jainism, Hinduism or their Point (1) is correct: As far as the sects. physical factors are concerned, it is clear Statement 2 is correct: Wood and that climate along with terrain and timber were used on a large scale since availability of water largely determines the advent of Vedic age. Skilled the pattern of the population distribution. craftsmen learnt to mimic texture, grain The North Indian Plains, deltas and and structure. Coastal Plains have higher proportion of Statement 3 is incorrect: Rock-cut population than the interior districts of chaitya shrines are present in 2 phases southern and central Indian States, of development, Hinyana (Theravada) Himalayas, some of the north eastern and Mahayana phase. and the western states. Source: Vajiram and Ravi booklet on Points (2) and (3) are correct: Among Indian Art and Culture, Volume 1, the socio-economic and historical factors Chapter 2, Page 39 and 43. of distribution of population, important ones are evolution of settled agriculture 31. U.P., Maharashtra, Bihar, West and agricultural development; pattern of Bengal, Andhra Pradesh along with human settlement; development of Tamil Nadu, Madhya Pradesh, transport network, industrialisation and Rajasthan, Karnataka and Gujarat, urbanisation. It is observed that the together account for about 76 per regions falling in the river plains and cent of the total population of the coastal areas of India have remained the country. On the other hand, share regions of larger population in India’s total population is very concentration. Even though the uses of small in the case of States like natural resources like land and water in Jammu & Kashmir, Arunachal these regions have shown the sign of Pradesh and Uttaranchal. Which of degradation, the concentration of the following factors are population remains high because of an responsible for such an uneven early history of human settlement and spatial distribution of population in development. India? Source: Geography NCERT - Class XII (1) Historical factors – India People and Economy; Chapter (2) Physical factors 1: Population: Distribution, Density, (3) Socio-economic factors Growth and Composition, Page no. 3 Select the correct answer using the code given below: 32. Consider the following statements (a) 1 only with reference to migration: (b) 2 only (1) Push factors cause people to (c) 2 and 3 only leave their place of residence (d) 1, 2 and 3 or origin. (2) Wars and local conflicts are Answer: (d) examples of pull factors. Explanation: Which of the statements given An uneven spatial distribution of above is/are correct? population in India suggests a close (a) 1 only relationship between population and (b) 2 only physical, socio-economic and historical (c) Both 1 and 2 factors. (d) Neither 1 nor 2

Prelim IAS Test Series (2019) – GS Test 20 (19.05.2019) 18 GS Comprehensive Test 2

Which of the statements given Answer: (a) below is/are correct? Explanation: (a) 1 only People, generally are emotionally (b) 2 only attached to their place of birth. But (c) Both 1 and 2 millions of people leave their places of (d) Neither 1 nor 2 birth and residence. There could be variety of reasons. These reasons can be Answer: (d) put into two broad categories: Explanation: (i) Push Factors: These cause people to Before the Partition Council, a Partition leave their place of residence or origin. In Committee was formed which was India people migrate from rural to urban chaired by Lord Mountbatten and its areas mainly due to poverty, high members were Vallabh Bhai Patel, Dr. population pressure on the land, lack of , Liaqat Ali Khan and basic infrastructural facilities like health Abdur Rab Nishtar. Later this Committee care, education, etc. Apart from these was replaced by a Partition Council. factors, natural disasters such as, flood, Hence, Statement 1 is incorrect. drought, cyclonic storms, earthquake, ● In this council, Congress was tsunami, wars and local conflicts also represented by Sardar Patel and Dr. give extra push to migrate. Hence, Rajendra Prasad, with C. Statement 1 is correct but Statement 2 Rajgopalachari as alternate member. is incorrect. ● Muslim league was represented by (ii) Pull Factors: These attract the Mohammed Ali Jinnah, Liaqat Ali people from different places. Pull factors Khan and Abdur Rab Nishtar as which attract people from rural areas to alternate member. cities. The most important pull factor for ● Even after 15 August 1947, this majority of the rural migrants to urban Partition Council was in existence, areas is the better opportunities, but the composition was changed as availability of regular work and relatively 2 members from each dominion. higher wages. Better opportunities for ● Patel and Dr. Prasad kept education, better health facilities and representing Indian Domain even at sources of entertainment, etc. are also that time. quite important pull factors. Sir Cyril Radcliffe was the head of a Source: Geography NCERT - Class XII Boundary Commission. Hence, – India People and Economy; Chapter Statement 2 is incorrect. Boundary 2 - Migration, Page no.20 Commission, consultative committee created in July 1947 to recommend how 33. Consider the following statements: the Punjab and Bengal regions of the (1) At the time of partition of Indian subcontinent were to be divided India, the Partition Council’s between India and Pakistan shortly Chairman was Sir Patrick before each was to become independent Spens, the ex-Chief Justice from Britain. The Commission— of India appointed by Lord Mountbatten, the final (2) An arbiter tribunal was set up viceroy of British India—consisted of four under the Presidentship of members from the Indian National Sir Cyril Radcliffe. Congress and four from the Muslim

Prelim IAS Test Series (2019) – GS Test 20 (19.05.2019) 19 GS Comprehensive Test 2

League and was chaired by Sir Cyril ax9BaoQ6AEwDHoECAgQAQ#v=onepa Radcliffe. ge&q=vidyut%20vahini&f=false Hence, the correct choice is option (d). 35. With reference to the Poona Pact, Source: consider the following statements: http://theconversation.com/how-the- (1) The idea of separate partition-of-india-happened-and-why- electorates for the Depressed its-effects-are-still-felt-today-81766 Classes was abandoned. (2) The seats reserved for the 34. In the context of Indian freedom Depressed Classes in both struggle, the organisation of an the Provincial and the army called “Vidyut Vahini” was Central Legislative associated with which of the Assemblies were increased. following? Which of the statements given (a) Prati Sarkar above is/are correct? (b) Tamralipti Jatiya Sarkar (a) 1 only (c) Hindustan Socialist (b) 2 only Republican Association (c) Both 1 and 2 (d) Swadesh Bandhab Samiti (d) Neither 1 nor 2

Answer: (b) Answer: (c) Explanation: Explanation: The people of Midnapore had played an The terms of the Poona Pact: important part in Quit India Movement. ● Seat reservation for the Scheduled Among the ‘National governments’ which Castes (SC) and Scheduled Tribes were formed in some areas was the (ST) in provincial legislature. Tamralipti Jatiya Sarkar formed at ● The STs and SCs would form an Tamluk in Bengal. electoral college which would elect ● It had its own army called Vidyut four candidates for the general Vahini and sister corps attached to it. electorate. This national government which ● The representation of these classes functioned within the limits laid down was based on the standards of joint by the Congress Committee kept on electorates and reserved seats. functioning till 8 August 1944. Hence, Statement 1 is correct. ● It was dissolved at the instance of ● About 19 percent of seats were to be Gandhi. It came into existence on reserved for these classes in 17th December, 1942 and undertook legislature. The system of election to cyclone reliefs works. the panel of candidates in both, Hence, the correct answer is option Central and Provincial Legislature (b). (Legislative Assemblies) should Source:https://books.google.co.in/books come to end in 10 years, unless it ?id=JT_qqzH3f3IC&pg=PA206&lpg=PA2 ends on mutual terms. Hence, 06&dq=vidyut+vahini&source=bl&ots=P6 Statement 2 is correct. gZ26WDJu&sig=ACfU3U3wKpY- ● The franchise for the Central and DtIn1Vsu0knlXXkFIigQCA&hl=en&sa=X Provincial Legislatures of these &ved=2ahUKEwi2t421uOXgAhVGqI8KH classes should be indicated in the Lothian Committee report.

Prelim IAS Test Series (2019) – GS Test 20 (19.05.2019) 20 GS Comprehensive Test 2

● There should be a fair representation Which of the statements given of these classes. above is/are correct? ● In every province, the SCs and STs (a) 1 only should be provided with sufficient (b) 2 only educational facilities. (c) 2 and 3 only Source: (d) 1, 2 and 3 https://www.indiatoday.in/education- today/gk-current-affairs/story/poona- Answer: (a) pact-338403-2016-09-24 Explanation: The Biennial Update Reports contains 36. Recently, world's longest salt cave information on national Greenhouse Gas was discovered. This salt cave is (GHG) inventories, including a national located in which of the following inventory report and information on countries? mitigation actions, needs and support (a) Israel received. Such reports provide updates (b) Egypt on actions undertaken by a Nation Party (c) Lebanon towards the cause of limiting (d) Syria Greenhouse gases emissions. Statement 1 is correct: BUR is Answer: (a) mandated under the United Nations Explanation: Framework Convention on Climate World's longest salt cave' has been Change. It was decided in 2012, recently discovered in Israel. Malham Conference of Parties (COP 17) of Cave in the Negev Desert is the longest UNFCCC that the first BURs from non- salt cave in the world, measuring more Annex I Parties to be submitted by than 10 kilometres in total length. The December 2014. The subsequent BURs 10 km of passages and chambers inside should be submitted every two years. Malham Cave, overlooking the Dead Statement 2 is incorrect: Flexibility is Sea, were mapped out over two years. given to the Least developed country Source: Parties (LDCs) and Small island https://www.bbc.com/news/world-middle- developing States (SIDS), which may east-47731943 submit such reports at their discretion. Statement 3 is incorrect: The India 37. Consider the following statements submission is done by the Climate with reference to 'Biennial Update Change Division of the Ministry of Reports (BUR)': Environment, Forest and Climate (1) They are mandated under Change (MoEF&CC). the United Nations Source:https://unfccc.int/process/transpa Framework Convention on rency-and-reporting/reporting-and- Climate Change. review-under-the-convention/biennial- (2) They are submitted by all the update-reports-and-international- countries party to the consultation-and-analysis-non-annex-i- Convention after every 2 parties/biennial-update-reports years. (3) The Ministry of New & 38. Consider the following statements Renewable Energy submits with reference to the Minamata the BUR for India. Convention:

Prelim IAS Test Series (2019) – GS Test 20 (19.05.2019) 21 GS Comprehensive Test 2

(1) It was signed during the Rio gold, silver and bronze according to the Earth Summit 1992. level of contributions received. (2) It covers mining and storage Sources: of mercury as well as  http://www.mercuryconvention.org/Co disposal of mercury waste. nvention/Text/tabid/3426/language/e Which of the statements given n-US/Default.aspx above is/are correct?  http://www.mercuryconvention.org/Co (a) 1 only nvention/History/HgClub/tabid/4222/l (b) 2 only anguage/en-US/Default.aspx (c) Both 1 and 2 (d) Neither 1 nor 2 39. Match List-I with List-II and select the correct answer using the code Answer: (b) given below: Explanation: List-I List-II The Minamata Convention on Mercury is (Location (Importa a global treaty to protect human health of nce) and the environment from the adverse Mauryan effects of mercury. era Statement 1 is incorrect: It was agreed artifacts) at the fifth session of the A. Nashik 1. History of Intergovernmental Negotiating Pandavlen Satvahan Committee on mercury in Geneva on i caves as 2013 and adopted later that year on 10 B. Naneghat 2. Grand October 2013 at a Diplomatic caves chaitya Conference (Conference of C. Bhaja 3. History of Plenipotentiaries), held in Kumamoto, caves tabla Japan. The Minamata Convention D. Karla 4. History of entered into force on 16 August 2017. caves numerals Statement 2 is correct: It calls for a ban Codes on new mercury mines, the phase-out of A B C D existing ones, the phase out and phase (a) 3 2 1 4 down of mercury use in a number of (b) 1 4 3 2 products and processes, control (c) 3 4 2 1 measures on emissions to air and on (d) 3 4 1 2 releases to land and water, and the regulation of the informal sector of Answer: (b) artisanal and small-scale gold mining. Explanation: The Convention also addresses interim Nashik-Pandavleni caves has an storage of mercury and its disposal once inscription ‘Nashikprashasti’ by Gautami it becomes waste, sites contaminated by Balasri, about achievements of mercury as well as health issues. Gautamiputra Satkarni, greatest of the Value Addition: “Mercury Club” was Satvahana rulers and his military established to recognize support to the victories. negotiating process for the legally Naneghat cave inscription have world’s binding instrument on mercury. It gives oldest known numeration symbols for ‘2, three different types of awards, namely,

Prelim IAS Test Series (2019) – GS Test 20 (19.05.2019) 22 GS Comprehensive Test 2

4, 6, 7 and 9’, that resembles modern teerthankar, the first four are attributed day Nagari numerals. to Parsvanatha, the 23rdteerthankar. Bhaja caves provide an important proof Hence, option (a) is the correct answer. regarding the history of tabla, or called Source: Vajiram and Ravi booklet on as Pushkara then; an Indian percussion Ancient History, Chapter 7, Page 44. instrument. Carvings from 200 BC show a woman playing tabla and another 41. Which of the following statements performing a dance. correctly describes “Crepuscular” Karla caves complex is best known for animals? ‘Grand Chaitya’, which is the largest and (a) They have no well-defined most completely preserved chaitya hall of Circadian Rhythm. the period. (b) They reside most of their Source: Vajiram and Ravi booklet on lives on trees. Indian Art and Culture, Volume 1, (c) They are active mainly during Chapter 2, Page 44, 45, 47 and 48. dawn and dusk. (d) They reside most of their 40. With reference to the Five Cardinal lives underground. Principles of Jainism philosophy, which of the following was not Answer: (c) given by Parsvanatha, the 23rd Explanation: teerthankar? Crepuscular animals are active mostly (a) Brahmacharya during dawn and dusk. For example: (b) Asteya Rabbits, deer, domestic house cat. (c) Aparigraha  Eusocial animals do not show any (d) Ahimsa well-defined sleep and wake patterns. Examples are honeybees at Answer: (a) early stages, naked mole rats. Explanation:  Arboreal animals spend majority of Jainism is considered world’s most their lives on trees. For example peaceful religion. Its main goal is the sloth, tree frog. liberation of the soul from the cycle of  Fossorial animals mostly live birth and death (Maya). To underground. For example: mole, achieve moksha or liberation of the soul, armadillo, badger. one must observe Five Cardinal Source:https://www.ncbi.nlm.nih.gov/pm Principles or Mahavratas. They are: c/articles/PMC3712434/ 1. Ahimsa or Ahinsa (Non-Violence) 2. Satya (Truth) 42. Consider the following statements: 3. Asteya or Achaurya (Not stealing) (1) In the Northern Hemisphere, 4. Aparigraha (Non-possessiveness) summer solstice occurs when 5. Brahmacharya (Celibacy) the Sun is directly over the These five great vows must be strictly Tropic of Capricorn, followed by the monks. There is some (2) In the Southern Hemisphere, concession in strictness of these vows winter solstice occurs when for the householders or lay followers, for the Sun is directly over the which they are referred to as anuvrata. Tropic of Cancer. Only the fifth principle ‘Brahmacharya’ Which of the statements given th was added by Mahavira, the 24 above is/are correct?

Prelim IAS Test Series (2019) – GS Test 20 (19.05.2019) 23 GS Comprehensive Test 2

(a) 1 only Source: G C Leong - Certificate (b) 2 only Physical and Human Geography, (c) Both 1 and 2 Chapter 1, The Earth and the (d) Neither 1 nor 2 Universe, Page 6.

Answer: (b) 43. Which of the following pairs are Explanation: correctly matched? A solstice is an event that happens twice Name Biological a year. It describes the moment that the Interactio Sun reaches its northern or n southernmost point. In essence, the Sun 1 Commensalis : Both is standing. More specifically, it occurs m species when the Sun's path is farthest from the benefit equator. 2 Mutualism : One Statement 1 is incorrect: The summer species solstice occurs when the Sun is directly benefits, over the Tropic of Cancer, which is other located at 23.5° latitude North, and runs unaffected through Mexico, the Bahamas, Egypt, 3 Amensalism : One Saudi Arabia, India, and southern China. species For every place north of the Tropic of unaffected Cancer, the Sun is at its highest point in , other the sky and this is the longest day of the species year. On the day of the summer solstice, suffers the Sun appears at its highest elevation 4 Parasitism : One with a noontime position that changes species very little for several days before and benefits, after the summer solstice. other Statement 2 is correct: The winter species solstice marks the shortest day and suffers longest night of the year. In the Northern Select the correct answer using the Hemisphere, it occurs when the Sun is code given below: directly over the Tropic of Capricorn, (a) 1 and 2 only which is located at 23.5° south of the (b) 1, 2 and 3 only equator and runs through Australia, (c) 1, 3 and 4 only Chile, southern Brazil, and northern (d) 3 and 4 only South Africa. In the Southern Hemisphere, it occurs when the Sun is Answer: (d) directly over the Tropic of Cancer. Explanation:  Commensalism: One species benefits while the other is not harmed. For example- Whales and Barnacles. Hence, pair (1) is incorrect.  Mutualism: When both the species benefits from the arrangement. For

Prelim IAS Test Series (2019) – GS Test 20 (19.05.2019) 24 GS Comprehensive Test 2

example – Algae and corals. Hence, draw water from the main water tank of pair (2) is incorrect. the town.  Amensalism: One organism inflicts ● Statement 1 is correct. In 1930, he harm to another organism without entered the national political scene. receiving any costs or benefits. For He demanded separate electorates example- Bread mould penicillin. for the untouchables. Hence, pair (3) is correct. ● Statement 2 is correct. In 1942, he  Parasitism: When one species started an organisation in Bombay derive the benefits at the cost of called Bahiskrit Hitkarni Sabha for the other species. For Example-Lice on moral and material upliftment of our head. Hence, pair (4) is correct. untouchables to enter the Hindu Source:https://www.ncbi.nlm.nih.gov/pm temple and draw water from public c/articles/PMC4942820/ wells. ● Statement 3 is correct. In 1942, he 44. Consider the following statements founded the Scheduled Caste about B.R. Ambedkar: Federation as an all India party. (1) He demanded separate Hence, the correct answer is (d). electorates for untouchables. Source: https://www.inc.in/en/our- (2) He started an organisation inspiration/dr-b-r-ambedkar called ‘Bahiskrit Hitkarni Sabha’ in Bombay. 45. Consider the following statements (3) In 1942, he founded the with respect to All India State Scheduled Caste Federation People’s struggle: as an all India party. (1) Nagpur session of Congress Which of the statements given in 1920 for the first time took above are correct? into account the demands of (a) 1 and 2 only state people’s movement. (b) 1 and 3 only (2) The All India State People’s (c) 2 and 3 only Conference was founded in (d) 1, 2 and 3 1927. Which of the statements given Answer: (d) above is/are incorrect? Explanation: (a) 1 only Babasaheb or Dr. Bhimrao Ramji (b) 2 only Ambedkar was a scholar, a social (c) Both 1 and 2 reformer and a leader who dedicated his (d) Neither 1 nor 2 life to eradicating social inequality in India. He established an India of equals, Answer: (d) a country which provided greater Explanation: opportunities for people who were The advance of the national movement in historically disadvantaged. By 1927, he British India, and the accompanying decided to launch active movements increase in political consciousness about against untouchability and espousing democracy, responsible government and access to public drinking water resources civil liberties had an inevitable impact on and the right to enter Hindu temples. He the people of the States. A much more led a satyagraha in Mahad to fight for the powerful influence was exercised by the right of the untouchable community to Non-Cooperation and Khilafat Movement

Prelim IAS Test Series (2019) – GS Test 20 (19.05.2019) 25 GS Comprehensive Test 2 launched in 1920; around this time and (1) Justice Pinaki Chandra under its impact, numerous local Ghose is the first anti- organizations of the State’s people came corruption ombudsman of the into existence. In some of the States country. Prajamandals or State’s People’s (2) The Lokpal Act provides for a Conferences were organized. chairperson and a maximum ● Statement 1 is correct. The policy of of eight members in the the towards Lokpal panel. the Indian states had been first (3) The chairperson and enunciated in 1920 at Nagpur when a members will hold office for a resolution calling upon the Princes to term of five years or until they grant full responsible government in attain 65 years of age. their States had been passed. (4) The jurisdiction of Lokpal ● Statement 2 is correct. In extends to the Members of December 1927, All India State’s Legislative Assemblies. People’s Conference (AISPC) was Which of the statements given convened and it was attended by 700 above are correct? political workers from the States. (a) 1 and 2 only Source: (b) 2 and 3 only http://cec.nic.in/wpresources/module/ (c) 1, 2 and 3 only History/PaperIV/4.7.10.2/e- (d) 1, 2 and 4 only content/aca%20%20and%20e- Content%204.7.10.2.doc Answer: (d) Explanation: 46. The primary duty of Rajukas,  Justice Pinaki Chandra Ghose is the appointed by Emporer Ashoka, first anti-corruption ombudsman of was of the country. Hence, statement 1 is (a) Propagation of Dhamma correct. (b) Dispensation of Justice  The Lokpal act provides for a (c) Tax collection Chairperson and a maximum of eight (d) Informants members in the Lokpal panel and of these, four need to be judicial Answer: (b) members. Hence, statement 2 is Explanation: correct. Within the empire, Asoka appointed a  The Chairperson and members will class of officers known as Rajukas, who hold office for a term of five years or were vested with authority of not only till they attain 70 years of age. rewarding people but also punishing Hence, statement 3 is incorrect. them, wherever necessary. So, they  The jurisdiction of Lokpal extends to administrated justice. Hence, option (b) the Prime Minister, Ministers, current is correct. and former Members of Parliament and Members of the Legislative 47. The Indian President appointed the Assemblies, government employees first anti-corruption ombudsman of and employees of companies funded the country. In this context consider or controlled by the Central or State the following statements: government. Hence, statement 4 is correct.

Prelim IAS Test Series (2019) – GS Test 20 (19.05.2019) 26 GS Comprehensive Test 2

Source: particular relevance for the livelihood of https://www.prsindia.org/media/article the landless and marginal farmers and s-by-prs-team/lokpal-act-3095 other weaker sections since many of them depend on income from their 48. Common Property Resource livestock due to the fact that they have denotes that limited access to land. CPRs also are (a) every member of the important for women as most of the community has the right of fodder and fuel collection is done by access and usage with them in rural areas. They have to devote specified obligations. long hours in collecting fuel and fodder (b) only Union government has from a degraded area of CPR. the right of access and usage Source: NCERT – India People and without any obligations. Economy, Chapter 5: Land Resources (c) every member of the and Agriculture, Page no. 43 community has the right of access and usage without 49. Consider the following map of any obligations. India: (d) only State government has the right of access and usage without any obligations.

Answer: (a) Explanation: Common Property Resources (CPRs) can be defined as community’s natural resource, where every member has the right of access and usage with specified obligations, without anybody having property rights over them. Community forests, pasture lands, village water bodies and other public spaces where a group larger than a household or family unit exercises rights of use and carries responsibility of management are examples of CPRs. Which of the following crops is Land, according to its ownership can grown in the shaded area marked broadly be classified under two broad with letter X as shown in the map heads – private land and common given above? property resources (CPRs). While the (a) Rice former is owned by an individual or a (b) Sugarcane group of individuals, the latter is owned (c) Jute by the state meant for the use of the (d) Wheat community. CPRs provide fodder for the livestock and fuel for the households Answer: (a) along with other minor forest products Explanation: like fruits, nuts, fibre, medicinal plants, India contributes 22 per cent of rice etc. In rural areas, such land is of production in the world and ranks second

Prelim IAS Test Series (2019) – GS Test 20 (19.05.2019) 27 GS Comprehensive Test 2 after China. About one-fourth of the total demand in international market due to its cropped area in the country is under rice superior quality. The iron ore mines cultivation. West Bengal, Punjab, Uttar occur in close proximity to the coal fields Pradesh, Andhra Pradesh and Tamil in the north-eastern plateau region of the Nadu were five leading rice producing country which adds to their advantage. states in the country in 2002-03. The Statement 3 is incorrect: About 95 per yield level of rice is high in Punjab, Tamil cent of total reserves of iron ore is Nadu, Haryana, Andhra Pradesh, West located in the States of Orissa, Bengal and Kerala. In the first four of Jharkhand, Chhattisgarh, Karnataka, these states almost the entire land under Goa, Andhra Pradesh and Tamil Nadu. rice cultivation is irrigated. Punjab and Source: NCERT Class XII – India Haryana are not traditional rice growing People and Economy, Chapter 7: areas. Rice cultivation in the irrigated Mineral and Energy Resources, Page areas of Punjab and Haryana was no 78 introduced in 1970s following the Green Revolution. 51. Consider the following statements Source: NCERT Class XII – India with respect to the Right to Justice: People and Economy, Chapter 5: (1) Guarantee of equal justice is Land, Resources and Agriculture, solely & explicitly mentioned Page no. 45 in the Preamble to the Constitution of India. 50. Consider the following statements (2) The New Delhi International with reference to the iron ore in Arbitration Centre Bill 2018 India: will boost the aim of Right to (1) India has the largest reserve Justice. of iron ore in Asia. Which of the statements given (2) Iron ore found in India is above is/are correct? mainly of inferior quality. (a) 1 only (3) Gujarat and Rajasthan (b) 2 only account for nearly 50% of (c) Both 1 and 2 Iron ore in India. (d) Neither 1 nor 2 Which of the statements given above is/are correct? Answer: (b) (a) 1 only Explanation: (b) 2 only Statement 1 is incorrect: The (c) 1 and 2 only guarantee of equal justice is meaningless (d) 1, 2 and 3 if the poor or illiterate or weak persons cannot enforce their rights because of Answer: (a) their poverty or illiteracy or weakness. It Explanation: is provided in both: the Preamble and Statement 1 is correct: India is the Directive Principles of the State endowed with fairly abundant resources Policy. of iron ore. It has the largest reserve of  Preamble: WE, THE PEOPLE OF iron ore in Asia. INDIA, having solemnly resolved to Statement 2 is incorrect: The two main constitute India into a SOVEREIGN types of iron ore found in our country are SOCIALIST SECULAR haematite and magnetite. It has great DEMOCRATIC REPUBLIC and to

Prelim IAS Test Series (2019) – GS Test 20 (19.05.2019) 28 GS Comprehensive Test 2

secure to all its citizens: JUSTICE, Scientific and Cultural social, economic and political etc. Organization.  Directive Principles of the State Select the correct answer using the Policy code given below: i. Articles 38 and 39 of the (a) 1 only Constitution of India lay down (b) 2 only clear mandate in this regard. (c) Both 1 and 2 According to Article 38 (1) the (d) Neither 1 nor 2 State shall strive to promote the welfare of the people by Answer: (d) securing and protecting as Explanation: effectively as it may a social order Statement 1 is incorrect. International in which justice, social, Vaccine Institute (IVI) is an international economic or political, shall non-profit organization devoted to inform all the institutions of the developing and introducing new and national life. improved vaccines to protect the people, ii. Article 39-A directs the State to especially children, against deadly ensure that the operation of the infectious diseases. legal system promotes justice on Statement 2 is incorrect. It was a basis of equal opportunity established in 1997 on the initiatives of and shall, in particular, provide the United Nations Development free legal aid by suitable Programme (UNDP). Its work is legislation or schemes or in any exclusively on vaccine development and other way, to ensure that introduction specifically for people in opportunities for securing justice developing countries, with a focus on are not denied to any citizen by neglected diseases affecting these reason of economic or other regions. disabilities. Statement 2 is correct: Key functions of 53. The Prime Minister’s Science, the NDIAC will include: (i) facilitating Technology and Innovation Council conduct of arbitration and conciliation in (PM-STIAC) has identified nine a professional, timely and cost-effective national missions to address major manner; and (ii) promoting studies in the scientific challenges to ensure field of alternative dispute India’s sustainable development. resolution. These are supposed to have Consider the following missions in a bearing on Right to Justice. this regard: Source: The Indian Constitution. (1) Natural Language Translation 52. Regarding the International (2) AGNIi Vaccine Institute (IVI), which of the (3) Agriculture following statements is/are correct? (4) Defence (1) It is an international non- Which of the missions above are profit organization devoted to not identified by the PM-STIAC? distribute vaccines free of (a) 1 and 2 only cost among poor people. (b) 3 and 4 only (2) It is an initiative of the United (c) 2 and 4 only Nations Educational, (d) None of the above

Prelim IAS Test Series (2019) – GS Test 20 (19.05.2019) 29 GS Comprehensive Test 2

The white-cheeked macaque is a species Answer: (b) of macaque found only in Mêdog County Explanation: in south-eastern Tibet and Arunachal  PM-STIAC is a 21-member advisory Pradesh in north-eastern India. The panel on science, technology and white-cheeked macaque was discovered innovation headed by Principal and described in 2015, making it one of Scientific Advisor to Union the most recently discovered mammals government. and one of the most recently discovered  It replaced earlier Scientific Advisory primates. Committee to Prime Minister and to Cabinet. 55. Consider the following statements  The council advises PM on science, with respect to the status of technology, as well as innovation. It workers and their rights: also coordinates implementation of (1) Article 23 of the Indian PMs scientific vision. Constitution prevents the  It actively aids in formulation and State from imposing timely implementation of major compulsory service for the science and technology missions and public purpose. evolve interdisciplinary technology (2) Unorganized Workers Social development programmes. Security Act 2008 includes all  PM-STIAC has identified nine the unorganized workers national missions to address major except the domestic workers. scientific challenges to ensure India’s Which of the statements given sustainable development. above is/are correct? Nine Missions Identified: Natural (a) 1 only Language Translation, Quantum Frontier, (b) 2 only Artificial Intelligence, National (c) Both 1 and 2 Biodiversity Mission, Electric Vehicles, (d) Neither 1 nor 2 Bioscience for Human Health, Waste to Wealth, Deep Ocean Exploration, AGNIi Answer: (d) Hence, option (b) is the correct Explanation: answer. Statement 1 is incorrect: Prohibition of Source: traffic in human beings and forced labour http://pibphoto.nic.in/documents/rlink/ (1) Traffic in human beings and begar 2019/mar/p20193601.pdf and other similar forms of forced labour are prohibited and any contravention of 54. White Cheeked Macaque is a this provision shall be an offence newly discovered primate species punishable in accordance with law (2) found in which of the following Nothing in this article shall prevent the States of India? State from imposing compulsory service (a) Mizoram for public purpose, and in imposing such (b) Arunachal Pradesh service the State shall not make any (c) Sikkim discrimination on grounds only of (d) Meghalaya religion, race, caste or class or any of them Answer: (b) Statement 2 is incorrect: The Central Explanation: Government is already implementing

Prelim IAS Test Series (2019) – GS Test 20 (19.05.2019) 30 GS Comprehensive Test 2

Unorganized Workers’ Social Security Act, 2008, to provide social security 57. A seven-year-old in Kerala has relating to life and disability cover, health been detected with the West Nile and maternity benefits, old age protection Virus. Consider the following to the unorganized workers including statements in this regard: domestic workers. (1) West Nile Virus is a viral Sources: infection resulting into a  http://pib.nic.in/newsite/PrintRelease. neurological disease. aspx?relid=187222 (2) The disease spreads through  https://economictimes.indiatimes.com mosquito bites. /news/economy/policy/new-national- (3) There are no vaccines to policy-for-domestic-workers-being- prevent or medications to drafted-40-lakh-to- treat the infection. benefit/articleshow/67419975.cms (4) It was first detected in a woman in the West Nile 56. With reference to the Index of district of Uganda. Industrial Production (IIP), which of Which of the statements given the following statements is/are above are not correct? correct? (a) 1 and 2 only (1) The IIP is compiled and (b) 2 and 3 only published annually by the (c) 3 and 4 only Department of Industrial (d) None of the above Policy & Promotion. (2) It covers manufacturing, Answer: (d) services and agriculture. Explanation: Select the correct answer using the  West Nile Virus is a viral infection code given below: which typically spread by mosquitoes (a) 1 only and results in neurological disease as (b) 2 only well as death in people. Hence, (c) Both 1 and 2 statements 1 and 2 are correct. (d) Neither 1 nor 2  The Virus is the member of the flavivirus genus and belongs to the Answer: (d) Japanese encephalitis antigenic Explanation: complex of the family Flaviviridae. Statements (1) and (2) both are  According to the World Health incorrect. Organisation (WHO). There are no The IIP is compiled and published every vaccines to prevent or medications to month by Central Statistics Office (CSO) treat the infection. Hence, statement of the Union Ministry of Statistics and 3 is correct. Programme Implementation. It covers  It was first detected in a woman in 682 items comprising Manufacturing (620 the West Nile district of Uganda in items), Mining (61 items) & Electricity (1 1937 and was later identified in birds item). The weights of the three sectors (crows and Columbiformes) in the are 75.53%, 14.16%, 10.32% Nile delta region in 1953. Hence, respectively on the basis of their share of Statement 4 is correct. GDP at factor cost during 2004-05. Sources:

Prelim IAS Test Series (2019) – GS Test 20 (19.05.2019) 31 GS Comprehensive Test 2

 https://indianexpress.com/article/w December 2009, the proposed START II hat-is/what-is-west-nile-virus- treaty, which never entered into force, kerala-kozhikode-5290762/ and the START III treaty, for which  https://www.nhp.gov.in/disease/co negotiations were never concluded. mmunicable-disease/west-nile- Under terms of the treaty, the number of fever strategic nuclear missile launchers will be  https://www.downtoearth.org.in/ne reduced by half. A new inspection and ws/health/seven-year-old-boy-dies- verification regime will be established, of-west-nile-virus-in-kerala-s- replacing the SORT mechanism. It does malappuram-63631 not limit the number of operationally inactive stockpiled nuclear warheads that 58. Consider the following statements: remain in the high thousands in both the (1) The New START Treaty is Russian and American inventories. between the United States of th America and the Russian 59. The Elections for the 17 Lok Federation on Measures for Sabha are being held in seven th the Further Reduction and phases from 11 of April 2019 to th Limitation of Strategic 19 May 2019. In this context, Offensive Arms. consider the following statements: (2) The New START replaced (1) A news channel publishes an the Treaty of Moscow exit poll after the elections to (SORT). the constituencies in the first Which of the statements given phase are over. above is/are correct? (2) A candidate makes appeal to (a) 1 only his followers through his (b) 2 only twitter account, a day before (c) Both 1 and 2 elections to come out from (d) Neither 1 nor 2 their homes and vote. (3) A candidate organizes a Answer: (c) musical concert at his own Explanation: house a day before election Statement 1 is correct. New START to attract the public and (Strategic Arms Reduction Treaty) is a amuse his supporters. nuclear arms reduction treaty between Which of the above could be seen the United States and the Russian as violation of Section 126/126A of Federation with the formal name of Representation of People act 1951 Measures for the Further Reduction and or the Model Code of Conduct? Limitation of Strategic Offensive Arms. It (a) 1 and 2 only was signed on 8 April 2010 in Prague, (b) 2 and 3 only and, after ratification, entered into force (c) 1 and 3 only on 5 February 2011. It is expected to last (d) 1, 2 and 3 at least until 2021. Statement 2 is correct. New START Answer: (c) replaced the Treaty of Moscow (SORT), Explanation: which was due to expire in December Statement 1 is correct (a violation of 2012. Its name is a follow-up to the section 126 of the RPA 1951) as: The START I treaty, which expired in Commission has clarified in the past that

Prelim IAS Test Series (2019) – GS Test 20 (19.05.2019) 32 GS Comprehensive Test 2 the said Section 126 prohibits displaying  http://pib.nic.in/newsite/PrintRelease. any election matter by means, inter alia, aspx?relid=99952 of television or similar apparatus, during  https://economictimes.indiatimes.com the period of 48 hours ending with the /news/politics-and- hour fixed for conclusion of poll in a nation/commission-plans-to-make- constituency. “Election matter” has been misuse-of-social-media-during-polls- defined in that Section as any matter a-civil- intended or calculated to influence or offence/articleshow/64008808.cms affect the result of an election. Violation of the aforesaid provisions of Section 126 60. Consider the following statements: is punishable with imprisonment upto a (1) Smartnet is an initiative of the period of two years, or with fine or both. Ministry of Electronics and Section 126A of the R.P. Act 1951, Information Technology conduct of Exit poll and dissemination of (MEITY) to support the their results during the period mentioned development of cities across therein, in the hour fixed for India. commencement of polls in the first phase (2) Smartnet provides a and half hour after the time fixed for horizontal learning and close of poll for the last phase in all the knowledge sharing platform States. for exchange between cities, Statement 2 is incorrect (not a practitioners, academia, violation of Model Code of Conduct or researchers and the RPA act) as: Under the current technologists. provisions of the model code of conduct Which of the statements given and RPA, there is no way to control the above is/are correct? election related matters on social media. (a) 1 only The election commission is yet to finalize (b) 2 only any guidelines regarding this matter. (c) Both 1 and 2 Statement 3 is correct (a violation of (d) Neither 1 nor 2 section 126 of the RPA 1951) as: The section 126 of prohibits a person to Answer: (b) propagate any election matter to the Explanation: public by holding, or by arranging the Statement 1 is incorrect. Smartnet is an holding of, any musical concert or any initiative of the Ministry of Housing and theatrical performance or any other Urban Affairs to support the development entertainment or amusement with a view of cities across India and to create a to attracting the members of the public resource-rich ecosystem of learning, thereto, in any polling area during the sharing and disseminating for city period of forty-eight hours ending with managers and primary stakeholders in the fixed for the conclusion of the poll for the urban transformation of India. any election in the polling area. Statement 2 is correct. Sources: The key objectives of Smartnet are:  http://www.legislative.gov.in/sites/def  Providing a horizontal learning and ault/files/04_representation%20of%2 knowledge sharing platform for 0the%20people%20act%2C%201951 exchange between cities, .pdf practitioners, academia, researchers and technologists;

Prelim IAS Test Series (2019) – GS Test 20 (19.05.2019) 33 GS Comprehensive Test 2

 Evolving a comprehensive framework destruction in outer space, not to visualise and articulate the ordinary weapons. Hence, government's urban sector missions Statement 3 is incorrect. such as smart cities, AMRUT,  The ASAT test was not directed Housing for All, HRIDAY and Swachh against any country. India’s space Bharat. capabilities neither threaten any country nor are they directed against 61. Recently, the DRDO conducted anyone. But as an added advantage Mission Shakti, an anti-satellite the capability achieved through the missile test, from Dr APJ Abdul anti-satellite missile test provides Kalam Island launch complex. With credible deterrence against threats to reference to ‘Mission Shakti’, which our growing space-based assets from of the following statements is/are long-range missiles and proliferation correct? in the types and numbers of missiles. (1) Under the mission, DRDO Hence, Statement 4 is incorrect. destroyed a live satellite in Source: the Low Earth Orbit. https://economictimes.indiatimes.com (2) India is only the third country /news/politics-and-nation/explained- after the U.S. and Russia to whats-mission-shakti-and-how-was-it- have the A-SAT technology. executed/articleshow/68607473.cms (3) Mission Shakti violates the 1967 Outer Space Treaty of 62. Consider the following statements: which India is a signatory. (1) The Data Security Council of (4) The test was mainly directed India (DSCI) is set up by the against the satellites of Ministry of Information and Pakistan and China. Broadcasting for data Select the correct answer using the protection in India. code given below: (2) The objective of DSCI is to (a) 1 only make the cyberspace safe, (b) 1, 2 and 3 only secure and trusted by (c) 1, 3 and 4 only establishing best practices in (d) 1, 2, 3 and 4 cyber security and privacy. Which of the statements given Answer: (a) above is/are correct? Explanation: (a) 1 only  DRDO-developed A-SAT system (b) 2 only successfully destroyed a live satellite (c) Both 1 and 2 in the Low Earth Orbit. Hence, (d) Neither 1 nor 2 Statement 1 is correct.  India is only the fourth country after Answer: (b) the U.S., Russia and China to have Explanation: the A-SAT technology. Hence, Statement 1 is incorrect and Statement 2 is incorrect. Statement 2 is correct.  Mission Shakti does not violate the Data Security Council of India (DSCI), is 1967 Outer Space Treaty of which a not-for-profit, industry body on data India is a signatory. The treaty protection in India, setup by NASSCOM, prohibits only weapons of mass committed to making the cyberspace

Prelim IAS Test Series (2019) – GS Test 20 (19.05.2019) 34 GS Comprehensive Test 2 safe, secure and trusted by establishing or harsh best practices, standards and initiatives criticism in cyber security and privacy. To further Codes its objectives, DSCI engages with A B C D governments and their agencies, (a) 3 2 4 1 regulators, industry sectors, industry (b) 1 4 3 2 associations and think tanks for policy (c) 1 2 3 4 advocacy, thought leadership, capacity (d) 3 1 2 4 building and outreach activities. Answer: (d) 63. Match the List-I with the List-II and Explanation: select the correct answer using the (1) Closure motion: Closure is one of code given below: the means by which debate may be List-I List-II brought to an end by a majority Type of Meaning decision of the House, even though Motion all Members wishing to speak have A. Closure 1 Certain rights not done so. At any time after a motion and motion has been made, any member immunities may move "That the question is now enjoyed by put". Such a motion is generally MPs, MLAs made at the conclusion of a speech and MLCs, but also at times whilst a member is individually addressing the House. and (2) Privilege motion: A privilege motion collectively is moved against a breach of B. Privilege 2 To draw the parliamentary privileges. motion attention of Parliamentary privileges are certain the House to rights and immunities enjoyed by a recent MPs, MLAs, and MLCs, individually matter of and collectively, so that they can public effectively discharge their functions. importance. When any of these rights and C Adjournm 3 A debate may immunities is disregarded, the ent motion be brought to offense is called a breach of privilege an end by a and is punishable under the law of majority Parliament or the state legislature. decision of (3) Adjournment motion: At the end of the House, the question-hour in the Parliament, even though motion moved by a member when it all Members is desired to draw the attention of the wishing to Executive for the purpose of speak have discussing a definite matter of urgent not done so public importance. D Censure 4 An (4) Censure motion: A motion which motion expression of seeks to censure the government for strong its “lapse”. If the motion is passed in disapproval the Popular House, the Cabinet

Prelim IAS Test Series (2019) – GS Test 20 (19.05.2019) 35 GS Comprehensive Test 2

resigns. A censure means an Statement 2 is correct. Only business- expression of strong disapproval or to-business transactions attract this levy. harsh criticism. It can be a stern rebuke by a legislature, generally 65. Recently, International Union for opposition against the policies of Conservation of Nature (IUCN) has Government or an individual minister. listed Hump-backed Mahseer as Sources: Critically Endangered in the Red  http://www.yourarticlelibrary.com/ List of Threatened Species. parliament/motions-in-india- Regarding ‘Hump-backed Mahseer’ parliament-six-motions/24894 which of the following statements  https://www.indiatoday.in/india/sto are correct? ry/the-two-motions-that-often-halt- (1) It is a species of freshwater parliament-proceedings-311235- crocodile. 2016-03-01 (2) It is referred to as tiger of the  https://rajyasabha.nic.in/rsnew/han water. dbook/chapter2-1.asp (3) It is found only in the Cauvery river basin. 64. With reference to ‘Google (4) The Government of India has tax/Equalization Levy’, sometimes been running ‘Project seen in the news, consider the Mahseer’ to enable following statements: conservation action for it. (1) It will be applied to payments Select the correct answer using the for online advertisements code given below: made by Indian business (a) 1 and 2 only entities to other Indian (b) 2 and 3 only advertisement companies. (c) 1, 2 and 4 only (2) Only business-to-business (d) 1, 2, 3 and 4 transactions attract this levy. Which of the statements given Answer: (b) above is/are correct? Explanation: (a) 1 only  The Hump-backed Mahseer is a (b) 2 only species of freshwater ray-finned fish. (c) Both 1 and 2 Hence, statement 1 is incorrect. (d) Neither 1 nor 2  It is referred to as tiger of the water. Hence, statement 2 is correct. Answer: (b)  The Hump-backed Mahseer is found Explanation: only in the Cauvery river basin Statement 1 is incorrect. Google tax / including Pambar, Kabini and Equalization Levy will apply to payments Bhavani rivers. Hence, statement 3 for online advertisements made by Indian is correct. business entities to non-residents (such  Shoal, an international as Google, Yahoo, Twitter, Facebook) organisation working to conserve where the aggregate payment in a freshwater species has initiated financial year to a non-resident exceeds ‘Project Mahseer’ in collaboration Rs 1 lakh. India became the first country with other stakeholders to enable to impose equalization levy, popularly conservation action for the hump- called as Google Tax.

Prelim IAS Test Series (2019) – GS Test 20 (19.05.2019) 36 GS Comprehensive Test 2

backed mahseer. Hence, statement Statement 2 is correct. Marrakesh 4 is incorrect. Treaty or Marrakesh VIP Treaty is Source: formally known as Marrakesh treaty to https://www.thehindu.com/sci- facilitate Access to Published works by tech/energy-and-environment/the- Visually Impaired Persons and Persons hump-backed-mahseer-critically- with Print Disabilities. It is also called endangered/article26653559.ece “Books for Blind” treaty. Statement 3 is incorrect. India is a 66. Consider the following statements: signatory to the UN Convention on the (1) The Sugamya Bharat Rights of Persons with Disabilities Abhiyan is a nation-wide (UNCRPD). flagship campaign to help visually impaired people 67. Which of the following taxes come especially students to study under the residuary power of maps. taxation as per the Indian (2) Marrakesh Treaty is to Constitution? facilitate access to published (1) Gift Tax works by visually impaired (2) Expenditure Tax persons and persons with (3) Sugarcane Cess print disabilities. (4) Entertainment Tax (3) India has not signed the UN Select the correct answer using the Convention on the Rights of code given below: Persons with Disabilities (a) 1 and 2 only (UNCRPD). (b) 1 and 3 only Which of the statements given (c) 1, 2 and 3 only above is/are correct? (d) 1, 2, 3 and 4 (a) 2 only (b) 1 and 2 only Answer: (c) (c) 1 and 3 only Explanation: (d) 3 only Article 248 of the Indian Constitution vests the residuary powers in the Answer: (a) Parliament. It says that Parliament has Explanation: exclusive power to make any law with Statement 1 is incorrect. On respect to any matter not enumerated in International day of persons with the Concurrent List or the State List. disability (3rd December), government Entry 97 in the Union List also lay down launched Accessible India Campaign that the Parliament has exclusive power (Sugamya Bharat Abhiyan) as a nation- to make laws with respect to any matter wide flagship campaign for achieving not mentioned in the State List or the universal accessibility for persons with Concurrent List including any tax not disabilities. The campaign targets three mentioned in either of these Lists. separate verticals for achieving universal Point (1) is correct: Decisions of the accessibility namely the built up High Courts were approved by the environment, transportation eco-system Supreme Court in Second Gift Tax and information & communication eco- Officer, Mangalore v. D. H. Nazareth. system. The Supreme Court had held that the Gift Tax Act was enacted by the Parliament

Prelim IAS Test Series (2019) – GS Test 20 (19.05.2019) 37 GS Comprehensive Test 2 and no Entry in the Union List and State Point (4) is incorrect: Entertainment tax List mentions such a tax. Therefore, is a type of tax which is levied by the Parliament purported to use its powers government on entertainment aspects derived from Entry 97 of the Union List like movie tickets, large scale commercial read with Article 248 of the Constitution. shows, and other private festival There is no other entry which covers the celebrations. The authorities responsible gift tax, the residuary powers of for the collection of entertainment tax Parliament were exercised to enact the from customers are the State law. governments. Point (2) is correct: The Andhra Sources: Pradesh High Court upheld the validity of  https://racolblegal.com/residuary- Expenditure Tax Act, 1957 as power-under-the-indian- expenditure tax which was not constitution-a-brief-analysis/ specifically provided for in any of the  https://www.coverfox.com/persona entries in List II or List III, was within the l-finance/tax/entertainment-tax/ ambit or scope of entry 97 of List I. So long as it was a tax on expenditure, the 68. Consider the following statements: mere fact that in furtherance of the (1) Under the Presumptive Tax legislative intent and object, the Scheme, the income is expenditure on which the tax was sought estimated on the basis of to be levied was not necessarily confined total turnover or gross to the expenditure actually incurred by receipts. the assesses himself (in this case it had (2) Under the scheme, the included the expenditure of his wife) did taxpayer needs to maintain not render it other than an expenditure detailed books of account tax. On appeal, the Supreme Court and get them audited. upheld the validity of the Expenditure Which of the statements given Tax Act, 1957 on the ground that it did above is/are correct? not fall within entry 62 List II, but under (a) 1 only the residuary powers. (b) 2 only Point (3) is correct: In Jaora Sugar (c) Both 1 and 2 Mills v. the State of M.P, the validity of (d) Neither 1 and 2 the central legislation was questioned. Gajendragadker C. J., delivering the Answer: (a) judgment of the Supreme Court held that Explanation: section 3 of the Central Act did not The presumptive taxation scheme was merely validate the invalid State Acts, framed and introduced by the Income because it would not have been Tax Act to ease the tax burden on small competent for Parliament to confer taxpayers who are involved in the jurisdiction on State Legislatures in that carrying out of any kind of business that way, but had included all the States and involves the leasing, renting out or plying Notifications in the Central Act at all of goods carriages. The scheme can be material times by virtue of section 3. used by businesses having a total Parliament had the power to levy the turnover of less than Rs 2 crore and cess as had been levied in the invalid eligible professionals with gross receipts State Acts, under Article 248 read with of less than Rs 50 lakh in a financial entry 97 of the List I. year.

Prelim IAS Test Series (2019) – GS Test 20 (19.05.2019) 38 GS Comprehensive Test 2

Statement 1 is correct. Under this  The programme of star rating of scheme, the eligible businesses can Microwave Ovens and Washing estimate their net income at the rate of Machines will be implemented on a 6% of the total turnover, if gross receipts voluntary basis and will be valid up to are received through digital mode of December 31, 2020. payments or at the rate of 8% in case of Source: cash receipts. https://www.thehindubusinessline.co Statement 2 is incorrect. Under the m/economy/microwave-ovens- scheme, the taxpayer need not to washing-machines-to-have-energy- maintain detailed books of account and efficiency- get them audited. rankings/article26475202.ece

69. In India we see star rating on 70. Consider the following statements: electrical appliances which depicts (1) Vostok 1 was the first their efficiency status. In this manned spaceflight of context consider the following mankind. statements: (2) Project Mercury was the (1) The rating of appliances in European program that put India is undertaken by the the first European astronauts Bureau of Energy Efficiency in space. (BEE). Which of the statements given (2) Microwave ovens and above is/are correct? washing machines will now (a) 1 only be assigned star ratings. (b) 2 only Which of the statements given (c) Both 1 and 2 above is/are correct? (d) Neither 1 nor 2 (a) 1 only (b) 2 only Answer: (a) (c) Both 1 and 2 Explanation: (d) Neither 1 nor 2 Statement 1 is correct. Vostok 1 was the first spaceflight of the Vostok Answer: (c) programme and the first manned Explanation: spaceflight in history. The Vostok 3KA  Star Rating of Appliances provides a space capsule was launched from basic sense of how energy efficient Baikonur Cosmodrome on April 12, 1961, each product is. Higher the rating, with Soviet cosmonaut Yuri Gagarin higher is the efficiency of the aboard, making him the first human to appliance. cross into outer space.  The rating of appliances in India is Statement 2 is incorrect. Project undertaken by the Bureau of Energy Mercury was the program by the NASA Efficiency (BEE). Hence, Statement that put the first American astronauts in 1 is correct. space in May 1961.  Electrical appliances microwave ovens and washing machines will 71. Which of the following is not a now be assigned star ratings based contempt of the Parliament of on their energy efficiency metrics. India? Hence, Statement 2 is correct.

Prelim IAS Test Series (2019) – GS Test 20 (19.05.2019) 39 GS Comprehensive Test 2

(a) Intimidation of the members Value Addition: Some of the other in connection with their important types of contempt of Parliamentary conduct. Parliament are:- (b) Giving false or misleading • Speeches or writings reflecting on the evidence or information House, its Committees or members; deliberately to the House. • Reflections on the character and (c) To prohibit the publication of impartiality of the Chairman/Speaker its debates and proceedings in the discharge of his duty; (d) Obstructing or molesting any • Publication of false or distorted report witness during his evidence of the Proceedings of the house; before a Committee of the • Publication oil expunged proceedings House. of the House; • Publication of proceedings of Secret Answer: (c) Sessions of the House; Explanation: • Pre-mature publication of Contempt of the House may be defined proceedings, evidence or report of a generally as "any act or omission which Parliamentary Committee; obstructs or impedes either House of • Reflections on the report of a Parliament in the performance of its Parliamentary Committee; functions, or • Molestation of Members on account which obstructs or impedes any Member of their conduct in the House or or officer of such House in the discharge obstructing Members while in the of his duty, or which has a tendency performance of their duties as directly or indirectly, to produce such Members or while on their way to or results." It may be stated that it is not coming after, attending the House or possible to enumerate exhaustively every a Committee thereof; act which might be construed by the • Offering bribes to Members to House as a contempt of the House. influence them in their Parliamentary Options (a), (b) and (d) are correct: conduct; • Intimidation of Members in • Any misconduct or undignified connection with their Parliamentary behaviour on the part of a Member, conduct; such as corruption in the execution of • Giving false or misleading evidence his office as Member, disorderly and or information deliberately to the undignified conduct contrary to the House or a Committee thereof, by a usage or inconsistent with accepted Member or a witness. standards of Parliamentary conduct; • Obstructing or molesting any witness • Obstructing or molesting officers of during his evidence before a the House in the discharge of their Committee of the House. duties; Option(c) is incorrect: To prohibit the Source: publication of its debates and https://rajyasabha.nic.in/rsnew/practic proceedings comes under the powers e_procedure/privileges.pdf necessary for the protection of its privileges and immunities to each House 72. Consider the following statements: of Parliament members. (1) The Drugs Technical Advisory Board (DTAB) is a statutory body under the

Prelim IAS Test Series (2019) – GS Test 20 (19.05.2019) 40 GS Comprehensive Test 2

aegis of the Ministry of (c) 2, 3 and 4 only Health & Family Welfare. (d) 1, 2, 3 and 4 (2) The DTAB advises only central government on Answer: (c) technical matters related to Explanation: the Drugs and Cosmetics  The World Happiness Report is Act, 1940. released by the United Nations Which of the statements given Sustainable Development Solutions above is/are correct? Network. Hence, statement 1 is (a) 1 only incorrect. (b) 2 only  The Report ranks countries on six (c) Both 1 and 2 key variables that support well-being: (d) Neither 1 nor 2 income, freedom, trust, healthy life expectancy, social support and Answer: (a) generosity. Hence, statement 2 is Explanation: correct. Statement 1 is correct and Statement  India has witnessed a sustained drop 2 is incorrect. with a 140th place this year The Drugs Technical Advisory Board is a compared with the 133rd place in statutory body under the Drugs and 2018. Hence, statement 3 is Cosmetics Act, 1940 to advice the correct. Central Government and the State  Finland topped the index of for the Governments on technical matters second year in a row. Hence, arising out of the administration of the statement 4 is correct. said Act and Rules made thereunder. Sources: The Board is headed by the Director  https://www.businesstoday.in/curr General of Health Services and DCGI ent/world/-happiness- acts as Member Secretary. ranking-drops-to-140-way-behind- pakistan-china- 73. With reference to the ‘World /story/330018.html Happiness Report 2019’, which of  https://www.thehindu.com/society/ the following statements are finland-is-the-happiest-nation- correct? india-slips-7-spots-ranks- (1) It is released by the United 140th/article26593703.ece Nations Development Programme (UNDP). 74. Recently, the Special Olympics (2) The Report ranks countries World Games 2019 were on six key variables. concluded at Abu Dhabi. In this (3) India has witnessed a context, which of the following sustained drop with a 140th statements is/are correct? place. (1) For the first time, the Special (4) Finland is the happiest Olympics were held in the country. Middle East and North Africa Select the correct answer using the region. code given below: (2) The Special Olympics are (a) 1 and 3 only held every four years. (b) 1, 2 and 4 only

Prelim IAS Test Series (2019) – GS Test 20 (19.05.2019) 41 GS Comprehensive Test 2

Select the correct answer using the (c) 2 and 3 only code given below: (d) 3 and 4 only (a) 1 only (b) 2 only Answer: (a) (c) Both 1 and 2 Explanation: (d) Neither 1 nor 2 When the bill is sent to the Governor after it is passed by the State Legislature, Answer: (a) he can a) Give his assent b) Withhold his Explanation: assent c) Return the bill (if its not a  The Special Olympics World Games money bill) for reconsideration of the 2019 were hosted in March 2019 by state legislature d) Reserve the bill for the cities of Abu Dhabi and Dubai in the consideration of the President. In one the United Arab Emirates. case such reservation is obligatory, that  For the first time, the Special is, where the bill passed by the state Olympics was held in the Middle East legislature endangers the position of the and North Africa region. Hence, state high court. In addition, the statement 1 is correct. Governor can also reserve the bill if it is  The Special Olympics are held every of the following nature: two years and alternates between  Ultra-vires, that is, Against the summer and winter. Hence, provisions of the Constitution statement 2 is incorrect.  Opposed to the Directive Principles Source: of State Policy https://www.khaleejtimes.com/nation/  Against the larger interest of the abu-dhabi/all-you-need-to-know- country about-special-olympics-world-games-  Of grave national importance abu-dhabi  Dealing with compulsory acquisition of property under Article 31A of the 75. In which of the following cases Constitution does the Governor’s reservation of Source: Vajiram and Ravi Yellow Book a bill for the President’s Indian Polity Part 2: Chapter 1 consideration becomes obligatory? Government of the State; page no 16 (1) If the bill passed by the State Legislature endangers 76. Arrange the following seas in order position of the State High of their location from North to Court. South on World Map: (2) If the bill is opposed to the (1) Caspian Sea Directive Principles of State (2) Red Sea Policy. (3) Baltic Sea (3) If the bill is dealing with Select the correct answer using the compulsory acquisition of code given below: property under Article 31A. (a) 1-2-3 (4) If the bill is against larger (b) 2-1-3 interests of the country. (c) 3-1-2 Select the correct answer using the (d) 1-3-2 code below: (a) 1 only Answer: (c) (b) 1 and 3 only Explanation:

Prelim IAS Test Series (2019) – GS Test 20 (19.05.2019) 42 GS Comprehensive Test 2

Caspian Sea: Caspian Sea is world’s (d) CITES largest inland body of water. It lies to the east of the Caucasus Mountains and to Answer: (b) the west of the vast steppe of Central Explanation: Asia. Earth Hour is an annual movement Red Sea: The Red Sea, one of the most organised by world Wildlife Fund under saline bodies of water in the world, is an which people, institutions, offices are inlet of the Indian Ocean between Africa encouraged to switch off non- essential and Asia. The connection to the ocean is lights for one hour from 8:30 to 9:30 pm. in the south through the Bab-el-Mandeb It is mainly held in March. It was started and the Gulf of Aden. The Sea has in 2007 at Sydney. played a crucial navigational role since Source:https://www.earthhour.org/what- ancient times. is-earth-hour Baltic Sea: The Baltic Sea is positioned in Northern Europe and bordered by 78. Consider the following statements Sweden (a part of the Scandinavian about the National Tiger Peninsula), Finland, Russia, Estonia, Conservation Authority (NTCA): Latvia, Lithuania, Poland, north-eastern (1) The NTCA is headed by the Germany, and eastern Denmark and its Minister of Environment numerous islands. Forest and Climate Change. (2) The NTCA has set up a Special Tiger Protection Force for protected areas. Which of the statements given above is/are correct? (a) 1 only (b) 2 only (c) Both 1 and 2 (d) Neither 1 nor 2

Answer: (c) Explanation: The National Tiger Conservation Authority (NTCA) is a statutory body with an overarching supervisory / coordination role, performing functions as provided in the Wildlife (Protection) Act, 1972. Statement 1 is correct: The Minister of Environment Forest and Climate Change Source: Orient BlackSwan School is the de- facto Chairman of NTCA. Atlas – Page no. 81 Statement 2 is correct: NTCA has created a Special Tiger Protection Force 77. Which of the following bodies to be stationed in selected protected organizes the Earth Hour? areas for the purpose. It will be funded by (a) IPCC NTCA and trained by specified (b) WWF paramilitary forces. (c) UNEP Value Addition:

Prelim IAS Test Series (2019) – GS Test 20 (19.05.2019) 43 GS Comprehensive Test 2

Under Project Tiger, 100% central Alexander captured Porus and, like the assistance is provided for non-recurring other local rulers he had defeated, items of expenditure to the States, allowed him to continue to govern his besides 50% matching grant for recurring territory. items (90% for North Eastern States), Source: Ancient India, R.S Sharma, based on the Annual Plan of Operations 11th class, Old NCERT page no. 76 of tiger reserve, proposed by the State vis-à-vis the Tiger Conservation Plan. 80. The capital city of which Source: Mahajanapada can be associated https://projecttiger.nic.in/content/38_1_Or with the modern city of Allahabad? ganizationalStructure.aspx (a) Kosala https://projecttiger.nic.in/content/37_1_Mi (b) Panchal lestoneInitiatives.aspx (c) Vatsa (d) Kuru 79. Consider the following statements regarding Alexander’s conquest in Answer: (c) India: Explanation: (1) Alexander entered the Indian Vatsa were of Kuru clan who shifted from sub-continent through the Hastinapur. They settled near confluence Bolan Pass. of Yamuna and Ganga. Kausambi was (2) Alexander crossed river Beas made their capital. The archaeological in order to defeat Porus. site of Kausambi is 40 km from city of Which of the statements given Allahabad. above is/are correct? Source: Ancient India, R.S Sharma, (a) 1 only 11th class, Old NCERT page no. 71 (b) 2 only (c) Both 1 and 2 81. Assertion (A): Twilight is longer at (d) Neither 1 nor 2 the Equator than that at the poles. Reason (R): The Sun rises and Answer: (d) sets almost at a vertical path at the Explanation: equator. Alexander entered the Indian sub- Given above are two statements continent through the Khyber Pass. labelled as Assertion (A) and Hence, Statement 1 is incorrect. Reason (R). In the context of the Porus was defeated by Alexander on the two statements, which of the banks of Jhelum. Alexander advanced following is correct? towards the river Beas only after (a) Both A and R is true and R is defeating Porus. Hence, Statement 2 is the correct explanation of A incorrect. (b) Both A and R is true but R is In 326 BC, Alexander invaded India, after not the correct explanation of crossing the river Indus he advanced A towards Taxila. He then challenged king (c) A is true but R is false Porus, ruler of the kingdom between the (d) A is false but R is true rivers Jhelum and Chenab. The Indians were defeated in the fierce battle, even Answer: (d) though they fought with elephants, which Explanation: the Macedonians had never seen before.

Prelim IAS Test Series (2019) – GS Test 20 (19.05.2019) 44 GS Comprehensive Test 2

The period between sunset and complete Source: darkness is twilight. The brief period https://indianexpress.com/article/india between sunrise and full daylight is /here-is-a-list-of-past-presidents-of- called dawn. The reason behind these indian-national-congress-4967084/ two phenomena is that the Earth receives diffused and refracted light from 83. Consider the following events the sun whilst it is still below the horizon. related to freedom struggle in India: Assertion statement is incorrect: (1) Second Round Table Twilight is longer at the poles than that at Conference the equator. At Poles, the Sun rises and (2) Launch of Individual civil sets in an oblique path which makes the disobedience period of refracted light received on Earth (3) The Government of India Act much longer. 1935 Reason Statement is correct: The Sun The events mentioned above took rises and sets almost at a vertical path at place during the tenure of which the equator, so the period during which Viceroy of India? the refracted light is received is short. (a) Lord Irwin Thus, the time the Sun takes to pass (b) Lord Willingdon through the twilight zone is relatively (c) Lord Linlithgow short as compared to the poles. (d) Lord Wavell Source: G C Leong Certificate Physical and Human Geography, Answer: (b) Chapter 1: The Earth and the Explanation: Universe, Pages 6 and 7. Events during tenure of Viceroy of India - Lord Willingdon (1931-1936): 82. Which of the following personalities  Second Round Table Conference has not served as the President of (1931) and failure of the Conference, the Indian National Congress presumption of civil disobedience. before the independence of India?  Announcement of Communal Award (a) 1932 under which separate (b) communal electorates were set up. (c) Sucheta Kripalani  “Fast unto Death” by Gandhi in (d) Nellie Sengupta Yeravada prison, broken after the Poona Pact (1932). Answer: (c)  Third Round Table Conference Explanation: (1932). Three female Presidents of Indian  Launch of individual Civil National congress before independence Disobedience Movement (1933). are as follows:  The Government of India Act (1935) ● Annie Besant: Calcutta Session, Dec. enacted. 28-29, 1917  Establishment of All India Kisan ● Sarojini Naidu: Kanpur Session, Dec. Sabha (1936) and Congress Socialist 26-28, 1925 Party by Acharya Narendra Dev and ● Nellie Sengupta: Calcutta Session, Jayprakash Narayan (1934). April 1, 1933  Burma-separated from India (1935). Hence, the correct answer is option (c).

Prelim IAS Test Series (2019) – GS Test 20 (19.05.2019) 45 GS Comprehensive Test 2

Hence, the correct answer is option History/PaperIV/4.7.10.2/e-content/ (b). aca%20%20and%20e- Source: A Brief History of Modern Content%204.7.10.2.doc India by Rajiv Ahir, Page no. 554 85. Consider the following statements 84. Which of the following leaders are regarding the ‘Circle of associated with the struggle of Illumination’: people in the Princely States? (1) The Circle of Illumination (1) Balwantrai Mehta always bisects the Earth’s (2) G. Subramaniya Iyer equator. (3) Maniklal Kothari (2) The Circle of Illumination Select the correct answer using the always bisects all the code given below: latitudes of the Earth. (a) 1 and 2 only Which of the statements given (b) 1 and 3 only above is/are correct? (c) 2 and 3 only (a) 1 only (d) 3 only (b) 2 only (c) Both 1 and 2 Answer: (b) (d) Neither 1 nor 2 Explanation: The advance of the national movement in Answer: (a) British India, and the accompanying Explanation: increase in political consciousness about The Circle of Illumination is the division democracy, responsible government and between day and night over the Earth. civil liberties had an inevitable impact on the people of the States. ● In the first and second decade of the twentieth century, runaway terrorists from British India seeking shelter in the States became agents of politicization. ● Some of the States in which prajamandals or States’ People’s Conferences were organized were Mysore, Hyderabad, Baroda, the Kathiawad States, the Deccan States, Jamnagar, Indore, and When the Earth rotates on its own axis, Nawanagar. only one portion of the Earth’s surface ● The men chiefly associated with the comes into the rays of the sun and struggle of people in the Princely experiences daylight. The other portion States were Balwantrai Mehta, away from the Sun will be in darkness. Maniklal Kothari and G.R. As the Earth rotates from West to East, Abhayankar. every part of the Earth’s surface will be Hence, option (c) is the correct under the Sun at some point or the other, answer. which explains the phenomena of sunrise Source: and sunset. http://cec.nic.in/wpresources/module/

Prelim IAS Test Series (2019) – GS Test 20 (19.05.2019) 46 GS Comprehensive Test 2

Statement 1 is correct: The circle of a licensing system. Each Party to the illumination always bisects the equator. Convention must designate one or more Statement 2 is incorrect: The circle of Management Authorities in charge of illumination bisects (cuts in half) all administering that licensing system. latitudes on the spring and autumnal Source: equinoxes. At this time, all places have https://www.cites.org/eng/disc/what.php equal day length. Source:http://www.earthonlinemedia.co 87. Consider the following statements m/ipg/outlines/lecture_earth_sun_relation about Nishkam Karma Math - a s.html social reform organisation in pre- independence times: 86. Consider the following statements (1) It was started by D.K. Karve. about CITES: (2) Educational progress of (1) It is funded from the corpus depressed classes was its of Global Environment Fund. prime objective. (2) It has a licensing system for (3) It founded a university in the international trade in Pune. specimens of selected Which of the statements given species. above are correct? Which of the statements given (a) 1 and 2 only above is/are correct? (b) 2 and 3 only (a) 1 only (c) 1 and 3 only (b) 2 only (d) 1, 2 and 3 (c) Both 1 and 2 (d) Neither 1 nor 2 Answer: (c) Explanation: Answer: (b) Nishkam Karma Math was established in Explanation: Pune in 1910. CITES (the Convention on International ● Statement 1 is correct. D.K. Karve Trade in Endangered Species of Wild founded the organisation. Fauna and Flora) is an international ● Statement 2 is incorrect. agreement between governments. Its Educational progress of women was aim is to ensure that international trade in its prime objective. specimens of wild animals and plants ● Statement 3 is correct. It also does not threaten their survival. founded a women’s university in Statement 1 is incorrect: The core Pune. administrative costs of the Secretariat, Value Addition: the Conference of the Parties and its About D.K. Karve – subsidiary bodies, the Standing ● Maharshi Dr. Committee and the other permanent (April 18, 1858 - November 9, 1962) committees, are financed from the CITES was a prominent social reformer of Trust Fund. This Trust Fund is his time in India in the field of welfare replenished from contributions from the of womankind. He promoted Parties to the Convention. education of women and freedom for Statement 2 is correct: Import and widows to remarry if they wished to export of species covered by the do so. The Government of India Convention, has to be authorized through recognized his reform work by

Prelim IAS Test Series (2019) – GS Test 20 (19.05.2019) 47 GS Comprehensive Test 2

awarding him in Indian rupees --a substantial sum in 1955 and highest civilian award, those days-- and the university was Bharat Ratna in 1958. then renamed as Shreemati Nathibai ● The work of Pandita Ramabai Damodar Thackersey Indian inspired Karve to dedicate his life to Women's University or SNDT the cause of female education, and Women's University. the work of Pandit Vishnushastri and ● In 1936, Karve started the Pandit Iswar Chandra Vidyasagar Maharashtra Village Primary inspired him to work for uplifting the Education Society with the goal of status of widows. Writings of Herbert opening primary schools in villages, Spencer had also highly influenced which had no schools run by the him. District Local Boards. He also ● In 1893, Karve founded Widhawa- encouraged maintenance of reading Wiwahottejak Mandali, which, habits of adults in villages. In 1944, besides encouraging marriages of he founded the Samata Sangh - widows, also helped the needy Association for the Promotion of children of widows. In 1895, the Human Equality. Besides dedicating institution was renamed as Widhawa- his life to the emancipation of women Wiwaha-Pratibandh-Niwarak Mandali in India, Karve stood for the abolition - Society to Remove Obstacles to of the caste system and the curse of Marriages of Widows. untouchability in the Hindu society. ● In 1896, Karve established a Hindu Karve wrote two autobiographical Widows' Home Association and works: Atmawrutta (1928) in Marathi, started in Hingane, a village then in and Looking Back (1936) in English. the outskirts of Pune Maharashtra, He ended the latter with the words: Mahilashram, a shelter and a school Here ends the story of my life. for women, including widows. Hence, the correct answer is option ● He started Mahila Vidyalaya in 1907; (c). the following year, he started Source: A Brief History Of Modern Nishkam Karma Math - Social India by Rajiv Ahirpg 572 Service Society to train workers for the Widows Home and the Mahila 88. Arrange the following major Vidyalaya. ‘Peasant Movements’ in ● After reading information about chronological order of their Japan Women's University in Tokyo, occurrence: Japan, Karve felt inspired to establish (1) Pabna Agrarian Uprising in 1916 in Pune the first university for (2) Ramoji Uprising women in India, with just five (3) Deccan Peasants Uprising students. The curriculum was tailored (4) Indigo Revolt to the aptitudes of women. Select the correct answer using the ● During 1917-1918, Karve established code given below: a Training College for Primary School (a) 4-1-3-2 Teachers and another school for (b) 1-2-4-3 girls, named Kanya Shal. In 1920, an (c) 2-4-1-3 industrialist and philanthropist from (d) 4-3-1-2 Mumbai, Sir Vithaldas Thackersey, donated Karve's university 1.5 million Answer: (a)

Prelim IAS Test Series (2019) – GS Test 20 (19.05.2019) 48 GS Comprehensive Test 2

Explanation: The correct order is: 89. Which of the following journals Indigo Revolt - 1859-60: It was a were brought under the aegis of peasant movement and subsequent Brahmo Samaj? uprising of indigo farmers against the (1) Sambad Kaumudi indigo planters that arose in Bengal in (2) Miratul Akbar 1859. The farmers were totally (3) Tattva Bodhini Patrika unprotected from the indigo planters, (4) Indian Mirror who resorted to mortgages or destruction Select the correct answer using the of their property if they were unwilling to code given below: obey them. Government rules favoured (a) 1 and 2 only the planters. By an act in 1833, the (b) 1 and 3 only planters were granted a free hand in (c) 1, 2 and 3 only oppression.[citation needed] Even the (d) 1, 2, 3 and 4 zamindars sided with the planters. Under this severe oppression, the farmers Answer: (d) resorted to revolt. Explanation: Pabna Agrarian Uprising 1873: It was a Journals associated with BrahmoSamaj: resistance movement against the ● Journals brought by Rammohan Roy: oppression of the zamindars. The Sambad Kaumudi (1821), Mirat-ul- zamindars routinely collected money Akhbar from the peasants by the illegal means of ● By Debendranath Tagore – forced levy, abwabs (cesses), enhanced Tattvabodhini Patrika rent and so on. Peasants were often ● By Keshub Chandra Sen - Indian evicted from land on the pretext of non- Mirror payment of rent. Value Addition: Deccan Peasants Uprising – 1875: In About Brahmo Samaj - May and June 1875, peasants of Raja Rammohan Roy founded the Maharashtra in some parts of Pune, Brahmo Sabha in August 1828; it was Satara and Ahmednagar districts later renamed, Brahmo Samaj. The revolted against increasing agrarian overall contribution of Brahmo Samaj distress. may be summed thus— (i) it denounced Phalke’s Ramoji Uprising – 1877: A polytheism and idol worship; (ii) it revolt was led by Vasudev Balwant discarded faith in divine avataras Phadke in 1877-78 in the Vashi & Panvel (incarnations); (iii) it denied that any area of Maharashtra. Phadke is known scripture could enjoy the status of as father of the armed struggle for India’s ultimate authority transcending/ human independence. He created a group called reason and conscience; (iv) it took no Ramoshi, which was the group of definite stand on the doctrine of karma Ramoshis, Kolis, Bhils and Dhangars and transmigration of soul and left it to- communities in Maharashtra and the individual Brahmos to believe either way; actually the “organized political dacoits“. (iv) it criticised the caste system. In He was captured and imprisoned in 1879 matters of social reform, the Samaj and died in 1883. attacked many dogmas and Hence the correct choice is option (a). superstitions. It condemned the Source: A Brief History Of Modern prevailing Hindu prejudice against going India by Rajiv Ahir pg 576-577 abroad. It worked for a respectable

Prelim IAS Test Series (2019) – GS Test 20 (19.05.2019) 49 GS Comprehensive Test 2 status for women in society—condemned credited with persuading hundreds of sati, worked for abolition, of purdah provinces to form India. system, discouraged child marriages and Value Addition: polygamy, crusaded for widow Gandhi Irwin Pact - On January 25, remarriage and for provisions of 1931 Gandhi and all other members of educational facilities, etc. It also attacked the Congress Working Committee (CWC) casteism and untouchability though in were released unconditionally. The CWC these matters it attained only limited authorised Gandhi to initiate discussions success. with the viceroy. As a result of these Hence, the correct choice is option discussions, a pact was signed between (d). the viceroy, representing the British Source: Indian Government, and Gandhi, https://www.thebrahmosamaj.net/hist representing the Indian people, in Delhi ory/history.html on February 14, 1931. This Delhi Pact, also known as the Gandhi-Irwin Pact, 90. Consider the following important placed the Congress on an equal footing resolutions passed by the Indian with the Government. Irwin on behalf of National Congress: the Government agreed on - 1. (1) Endorsement of Gandhi-Irwin immediate release of all political pact prisoners not convicted of violence; 2. (2) Resolution on Fundamental remission of all fines not yet collected; 3. Rights return of all lands not yet sold to third (3) Endorsement of National parties; 4. lenient treatment to those Economic Programme government servants who had resigned; Who was the President of Indian 5. right to make salt in coastal villages for National Congress Session in personal consumption (not for sale); 6. which decisions mentioned above right to peaceful and non-aggressive were taken? picketing; and 7. withdrawal of (a) emergency ordinances. The viceroy, (b) Netaji Subhash Chandra however, turned down two of Gandhi's Bose demands— (i) public inquiry into police (c) Dr. Rajendra Prasad excesses, and (ii) commutation of Bhagat (d) Sardar Singh and his comrades' death sentence to life sentence. Gandhi on behalf of the Answer: (d) Congress agreed— (i) to suspend the Explanation: civil disobedience movement, and (ii) to Sardar Vallabhbhai Patel was president participate in the next Round Table of the Karachi session in 1931. Conference on the constitutional ● The conference endorsed the question around the three Finch-pins of Gandhi-Irwin Pact under his federation, Indian responsibility, and Presidentship at this session. reservations-and safeguards that may be ● Resolution on Fundamental Rights necessary in India's interests (covering and Endorsement of National such areas as defence, external affairs, Economic Programme were also position of minorities, financial credit of carried out. India and discharge of other obligations). Patel was one of the leading figures of The resolution on Fundamental Rights Indian independence movement and is guaranteed - * free speech and free

Prelim IAS Test Series (2019) – GS Test 20 (19.05.2019) 50 GS Comprehensive Test 2 press * right to form associations * right Select the correct answer using the to assemble * universal adult franchise * code given below: equal legal rights irrespective of caste, (a) 1 and 2 only creed and sex * neutrality of state in (b) 1 and 3 only religious matters * free and compulsory (c) 2 and 3 only primary education * protection to culture, (d) 3 only language, script of minorities and linguistic groups. Answer: (d) The resolution on National Economic Explanation: Programme included - * substantial Seismic discontinuities are detected on reduction in rent and revenue * the basis of velocity of seismic waves, exemption from rent for uneconomic which vary as they pass through holdings * relief from agricultural materials of different densities. indebtedness * control of usury * better Pair 1 is incorrect: Gutenberg conditions of work including a living discontinuity is the contact zone of the wage, limited hours of work and mantle and the outer core. Seismic protection of women workers * right to waves recorded at increasing distances workers and peasants to form unions * from an earthquake indicate that seismic state ownership and control of key velocities (speeds) gradually increase industries, mines and means of transport. with depth in the mantle. However, at arc Hence, the correct answer is option distances of between about 103° and (d). 143° no P waves are recorded. Source: https://indianexpress.com/article/india /here-is-a-list-of-past-presidents-of- indian-national-congress-4967084/

91. Which of the following pairs is/are correctly matched? Seismic Between Discontinuity 1. Gutenberg : Contact Discontinuity zone of Furthermore, no S waves are record the outer beyond about 103°. In 1914 Gutenberg core and explained this as the result of a molten the inner core beginning at a depth of around 2900 core km. Shear waves could not penetrate this 2. Lehman’s : Contact molten layer and P waves would be Discontinuity Zone of severely slowed and refracted (bent). the Outer Gutenberg helped discover the boundary core and between the mantle and the core. Mantle Pair 2 is incorrect: Lehman’s 3. Moho : Contact Discontinuity is the contact zone between Discontinuity Zone of the outer core and the inner core. the Crust Between 143° and 180° from an and the earthquake another refraction (bending) Mantle is recognized (Lehman, 1936) resulting

Prelim IAS Test Series (2019) – GS Test 20 (19.05.2019) 51 GS Comprehensive Test 2 from a sudden increase in P wave Answer: (b) speeds at a depth of 5150 km. This Explanation: speed increase is consistent with a Anti-satellite (ASAT) weapons are change from a molten outer core to a missile-based systems to attack moving solid inner core. Lehman helped discover satellites. So far the United States, that there was not only a liquid iron core, China, India and Russia are the only but that there were actually two cores: countries who’ve reported the ability to the outer liquid core and the inner solid shoot down space objects from ground or core. airborne sources. The development of Pair 3 is correct: Moho discontinuity is such systems has a long history — the contact zone of the mantle and the fuelled by the Cold War between the crust. The depth to the Moho beneath the United States and the former Soviet continents averages around 35 km but Union — with a waxing or waning of ranges from around 20 km to 70 km. The funding. There are different kinds of Moho beneath the oceans is usually systems — those that can be launched about 7 km below the seafloor (i.e., from the ground or those vaulted from ocean crust is about 7 km thick). This is a planes. depth where seismic waves change The success of ‘Mission Shakti,’ velocity and there is also a change in demonstrated India’s anti-satellite missile chemical composition. Also termed the capability by shooting down a live Mohorovicic; discontinuity after the satellite. While 'Mission Shakti' may have Croatian seismologist Andrija targeted an object in outer space, India Mohorovicic; (1857-1936) who has long developed the ability to discovered it. intercept incoming missiles. Source: Source:https://www.thehindu.com/news/ https://www.ucl.ac.uk/EarthSci/people/lid national/the-hindu-explains-what-is- unka/GEOL2014/Geophysics7%20- significant-about-mission- %20Deep%20Earth/Earth%20Structure.h shakti/article26652980.ece tm https://seismicdiscontinuity.weebly.com/g 93. Consider the following statements utenberg-discontinuity.html about ‘Ecological Task Force’: (1) Its battalions are raised 92. Which of the following countries under the Territorial Army. have reported the ability to shoot (2) It is an armed battalion down space objects like satellites against poaching and from ground? trafficking. (1) China Which of the statements given (2) Russia above is/are correct? (3) France (a) 1 only (4) Israel (b) 2 only Select the correct answer using the (c) Both 1 and 2 code given below: (d) Neither 1 nor 2 (a) 2 only (b) 1 and 2 only Answer: (a) (c) 2 and 3 only Explanation: (d) 1, 2 and 4 only Dr Norman Borlough, from Wheat and Maize Centre, Mexico suggested to the

Prelim IAS Test Series (2019) – GS Test 20 (19.05.2019) 52 GS Comprehensive Test 2 former Prime Minister of India, Late Mrs. Source:https://www.thehindu.com/sci- to involve the Indian Army tech/technology/pslv-isro-emisat-launch- to restore the ecological balance on a from-sriharikota-on-april- war footing. 1/article26699077.ece Statement 1 is correct: As the regular Army could not be utilised for the 95. Arrange the following kingdoms purpose, it was decided that a Territorial chronologically from earliest to Army Unit be raised for the purpose, latest as annexed by the British Raj enrolling ex-servicemen from the region. under the pretext of ‘Doctrine of Statement 2 is incorrect: Main Lapse’: functions of Ecological Task Force (1) Jhansi involve afforestation and protecting (2) Satara against land degradation. (3) Nagpur Presently nine such Infantry Battalions (4) Baghat (TA) Ecological, comprising 22 Select the correct answer using the operational companies, are carrying out code given below: afforestation in difficult and degraded (a) 3-1-2-4 areas in States of Uttarakhand, (b) 1-2-3-4 Rajasthan, J & K, Himachal (c) 2-4-1-3 Pradesh, Delhi, Assam and Maharashtra. (d) 3-4-2-1 Source: https://www.territorialarmy.in/ecological.a Answer: (c) spx Explanation: Doctrine of lapse, in Indian history, is a 94. Which of the following is India’s formula devised by Lord Dalhousie, first electronic surveillance satellite, governor-general of India (1848–56), to recently launched by ISRO? deal with questions of succession to (a) Resourcesat Hindu Indian states. It was a corollary to (b) PiSat the doctrine of paramountcy, by which (c) EMIsat Great Britain, as the ruling power of the (d) SRMsat Indian subcontinent, claimed the superintendence of the subordinate Answer: (c) Indian states and so also the regulation Explanation: of their succession. The Indian Space Research Organisation Annexation in the absence of a natural or (ISRO) launched the country’s first adopted heir was enforced in the cases electronic surveillance satellite, EMISAT, of Satara (1848), Jaitpur and Sambalpur from Sriharikota in coastal Andhra (1849), Baghat (1850), Chota Udaipur Pradesh. EMISAT is a satellite built (1852), Jhansi (1853), and Nagpur around ISRO’s Mini Satellite-2 bus (1854). They have generally been weighing about 436 kg. The satellite was regarded as having contributed to the successfully placed in its intended sun- discontent that was a factor in the synchronous polar orbit of 748 km height outbreak (1857) of the Indian Mutiny and by PSLV-C45 on April 01, 2019. The the widespread revolt that followed. satellite is intended for electromagnetic Source: Vajiram and Ravi: Yellow spectrum measurement. Book - Modern Indian History Yellow Book Pg 124-125

Prelim IAS Test Series (2019) – GS Test 20 (19.05.2019) 53 GS Comprehensive Test 2

 To inculcate political awareness and 96. With reference to the Indian spirit of nationalism. Hence, National Congress, consider the objective (1) is correct. following:  Provide meeting grounds for (1) To inculcate political exchange of ideas. awareness and spirit of  To draft and petition government to nationalism. introduce popular reform. Hence, (2) To draft and petition objective (2) is correct. government to introduce  To develop and propagate an anti- popular reform. colonial nationalist ideology. (3) To establish a headquarters  To establish headquarter for for the national movement. movement. Hence, objective (3) is (4) To garner sentiment of correct. sympathy in England towards  Later, it was that political ideas, patriotic aspiration of Indians. ideology and activity of leadership of (5) To push forward demand for Indian National Congress should dominion status for India. have some recognition in England. Which of the above were objectives spearheaded this of the Indian National Congress in campaign. Hence, objective (4) is its early years of formation? correct. (a) 1 and 2 only  There was no demand of dominion (b) 1, 2 and 4 only status for India in early days of Indian (c) 1, 2, 3 and 4 only National Congress. Hence, objective (d) 1, 2, 3 and 5 only (5) is incorrect. Sources: Vajiram & Ravi, Yellow Book Answer: (c) - Modern Indian History, Page 138 Explanation: The Indian National Movement got an 97. With reference to decentralization impetus after the foundation of the Indian under the British, consider the National Congress in 1885. Mr. A.O. following statements: Hume, a retired member of the Indian (1) Mayo’s Resolution was Civil Service, took the initiative in this limited to financial direction. He appealed to the Graduates decentralization only. of the Calcutta University to form an (2) Ripon’s Resolution aimed for association for the moral, social and training the Indians in the art political regeneration of the Indians. of governance at local level. Aims and Objectives of the Congress Which of the statements given Leaders from 1885 to 1905: The early above is/are correct? phase of the National Movement was (a) 1 only dominated by the moderate leaders like (b) 2 only Dada Bhai Nauroji, Surendranath (c) Both 1 and 2 Banerjee, Ferozshah Mehta, Gopal (d) Neither 1 nor 2 Krishna Gokhale etc. They believed in peaceful and constitutional methods. Answer: (c) They largely focused on: Explanation:

Prelim IAS Test Series (2019) – GS Test 20 (19.05.2019) 54 GS Comprehensive Test 2

Statement 1 is correct. In 1870, the Viceroy of India, Lord Mayo took a 98. Consider the following statements: Resolution to bring about administrative (1) Singba Rhododendron efficiency in meeting the demands of the Sanctuary is located in this people and to add to the finances of the state. ‘existing imperial resources which will not (2) Rhododendron Nevium is suffice for the growing wants of the designated as its 'State Tree'. country'. Though out of a fiscal The description given above is compulsion, Mayo's Resolution on local most suitable for which of the self-government was to be adopted, still, following States of India? it was a landmark in the evolution of the (a) Sikkim British regime's policy in the sphere of (b) Arunachal Pradesh decentralization. It was limited to the (c) Assam financial domain. (d) Manipur Statement 2 is correct. In India, democratic decentralization as a means Answer: (a) to political development was the guiding Explanation: philosophy of Lord Ripon's resolution of Singba Rhododendron Sanctuary is 1882. He was first to initiate situated in the state of Sikkim. decentralization discourse during the Rhododendron Nevium is the State Tree British imperial rule. Ripon pleaded for of Sikkim. Rhododendron is a species of developing local self-government for the flowers which grows on trees and shrubs. purpose of training the Indians in the art Source: of governance. Ripon wanted to develop www.sikkimforest.gov.in/docs/IBA/sk11.p local government institution as a means df for effecting improvement in administration as well as an instrument of 99. Consider the following statements political and popular education of the with reference to 'Electoral Bonds': people. Thus, his resolution stood for (1) These bonds can be decentralization of a large network of purchased at specified local Self-governing bodies. He branches of the Union Bank emphasized that local bodies should of India (SBI). have- a) elected non-official majority: b) (2) These bonds may be bought non-official chairman; and c) elastic and for any value, in multiples of sufficient financial resources. He was ₹2,000, ₹20,000, ₹2 lakh, against the government's control over the ₹20 lakh or ₹2 crore. local self-governing bodies to be Which of the statements given exercised from without. Ripon wanted for above is/are correct? a) Training the Indian people in the art of (a) 1 only governance, b) enabling them to learn (b) 2 only from experience, and c) Opening up (c) Both 1 and 2 avenues for political participation of (d) Neither 1 nor 2 educated people, urged for an early establishment of the local boards not Answer: (d) only in cities but also in rural areas. Explanation: Sources: A brief History of Modern The Electoral Bond Scheme was India, Spectrum’s, page 253-254 launched in an attempt to “cleanse the

Prelim IAS Test Series (2019) – GS Test 20 (19.05.2019) 55 GS Comprehensive Test 2 system of political funding in the country.” (3) There is a legislative control A donor may buy an electoral bond at over the use of funds from specified banks and branches using the Public Account of India. electronic modes of payment and after Which of the statements given having completed the KYC (know your above is/are not correct? customer) requirements. The political (a) 1 only party will have to deposit the encashed (b) 1 and 3 only money in a bank account it has informed (c) 2 and 3 only the Election Commission about. (d) 1, 2 and 3 Statement 1 is incorrect: The bonds are available at specified branches of the Answer: (c) State Bank of India (SBI) for 10 days Explanation: each in the months designated by the Public Account of India funds being government from time to time. managed by the Government is a Statement 2 is incorrect: The bonds perpetual source of distortion in our may be bought for any value, in multiples public finances. It is time to separate all of ₹1,000, ₹10,000, ₹1 lakh, ₹10 lakh or Public Account funds from the cash ₹1 crore. The State Bank of India issued balances of the Government and entrust a whopping ₹1,716 crore in electoral their management to professional trusts bonds in just two months of 2019. The free from Government control. RTI replies say electoral bonds worth Statement 1 is correct: According to ₹495.6 crore were sold in Mumbai in Article 266, all other public money 2019, which is the largest share (28.9%) received by or on behalf of the of the total amount sold. Government of India or the Government Source: of a State shall be entitled to the Public https://www.thehindu.com/news/national/ Account of India or the Public Account of have-electoral-bonds-reduced- the State. anonymous-cash- Statement 2 is incorrect: Part XII of the donations/article26757199.ece Indian Constitution deals with the government finance which is organized 100. In the context of the 'Funds of under three funds. Under article 266(1), Government of India', consider the all revenues received by the Government following statements about the of India or any state, all loans raised by Public Account of India: the issue of treasury bills, loans or (1) According to Article 266 of ways and means advances and all the Constitution, all other money received by the Government in public money received by or repayment of loans shall form the on behalf of the Government Consolidated Fund of India. of India shall be entitled to Statement 3 is incorrect: Public the Public Account of India. Account, which does not involve (2) All revenues received by the revenues or debt of the Governments but Government and all loans other public money that does not belong raised by the issue of to the Government as such. Thus there is treasury bills come under the no legislative control over the use of Public Account of India. funds from the Public Account, and it is this lack of legislative control that makes the article 266(2) somewhat

Prelim IAS Test Series (2019) – GS Test 20 (19.05.2019) 56 GS Comprehensive Test 2 intriguing, and this is what also makes it vulnerable to misuse and subject to many aberrations in our financial system. Source: http://yojana.gov.in/public- account.asp

Prelim IAS Test Series (2019) – GS Test 20 (19.05.2019) 57 GS Comprehensive Test 2